You are on page 1of 69

www.VNMATH.

com

PHNG TRNH HM - K THUT GII V


MT S VN LIN QUAN
Trn Minh Hin - GV trng THPT chuyn Quang Trung, Bnh Phc
Ngy 15 thng 6 nm 2011

Mc lc
Mc lc

1 Phng php th bin

2 Phng trnh hm Cauchy

12

3 Phng php quy np

19

4 Khai thc tnh cht n nh, ton nh, song nh, chn l ca hm s

24

5 Khai thc tnh n iu ca hm s

34

6 Khai thc tnh cht im bt ng ca hm s

40

7 Phng php a v phng trnh sai phn

44

8 Phng php s dng tnh lin tc ca hm s

46

9 ng dng phng trnh hm c bn

53

10 Bt ng thc hm

60

11 Hm tun hon

65

12 Mt s chuyn phng trnh hm


66
12.1 Phng trnh hm gii nh tnh gi tr hm s theo hai cch khc nhau . . . . . . . . . . 66
13 Gii phng trnh hm bng cch thm bin

68

14 LUYN TP PHNG TRNH HM


69
14.1 Phng php th bin . . . . . . . . . . . . . . . . . . . . . . . . . . . . . . . . . . . . . 69
14.2 Bt ng thc hm . . . . . . . . . . . . . . . . . . . . . . . . . . . . . . . . . . . . . . . 69

www.VNMATH.com

1 PHNG PHP TH BIN

Phng php th bin

Phng php th bin c l l phng php c s dng nhiu nht khi gii phng trnh hm. Ta
c th:
Hoc cho cc bin x, y, . . . nhn cc gi tr bng s. Thng cc gi tr c bit l 0, 1, 2, . . .
Hoc th cc bin bng cc biu thc lm xut hin cc hng s hoc cc biu thc cn thit.
Chng hn, nu trong phng trnh hm c mt f (x + y) m mun c f (0) th ta th y bi x,
mun c f (x) th cho y = 0, mun c f (nx) th th y bi (n 1)x.
V d 1.1. (o 199?) Tm tt c cc hm s f : R R tha mn iu kin
x2 f (x) + f (1 x) = 2x x4 , x R.
Gii
Thay x bi 1 x ta c
(1 x)2 f (1 x) + f (x) = 2(1 x) (1 x)4 , x R.
Nhu vy ta c h

8
<

x2 f (x) + f (1 x) = 2x x4
:f (x) + (1 x)2 f (1 x) = 2(1 x) (1 x)4

Ta c D = (x2 x 1) (x2 x + 1) v Dx = (1 x2 ) (x2 x 1) (x2 x + 1). Vy D.f (x) = Dx , x


R. T ta c nghim ca bi ton l
8
>
>
<

1 x2
: x 6= a, x 6= b,
(c l hng s ty ),
f (x) = >c R
: x = a,
>
:
2a a4 a2 c : x = b,
vi a, b l nghim ca phng trnh x2 x 1 = 0.
Nhn xt: Bi ton trn c dng mt ln na trong k thi VMO 2000, bng B.
V d 1.2. Tm tt c cc hm s f : R R tha mn iu kin
f (x + y) + f (x y) = 2f (x) cos y, x, y R
Hint: 1. Th y 2
2. Th y y + 2 hoc th x = 2
3. Th x 0
p s: f (x) = a cos x + b sin x(a, b R)
V d 1.3. f : R R tha mn iu kin f (xy + x + y) = f (xy) + f (x) + f (y), x, y R. Chng minh
rng:
f (x + y) = f (x) + f (y), x, y R.
GV: Trn Minh Hin . . . . . . PTH bi dng hc sinh gii . . . . . . Trng THPT chuyn Quang Trung

www.VNMATH.com

1.
2.
3.
4.
5.

1 PHNG PHP TH BIN

Hint:
Tnh f (0)
Th y = 1, chng minh f l hm l
Th y = 1 f (2x + 1) = 2f (x) + 1
Tnh f (2(u + v + uv) + 1) theo (3) v theo gi thit suy ra f (2uv + u) = 2f (uv) + f (u)
Cho v = 21 , u2 x v u y, 2uv x suy ra iu phi chng minh

V d 1.4. Tm tt c cc hm s f : R R ng thi tha mn cc iu kin sau:




1
, x 6= 0
x
f (x) + f (y) = 1 + f (x + y), x, y R, (x, y) 6= (0, 0); x + y 6= 0
f (x) = xf

Hint:
1. Tnh f (0), f (1)

1
2. Tnh a + 1 vi a = f (1) = f x+1
= f x + 1 x+1
theo c hai iu kin.
x+1
p s: f (x) = x + 1
Nhn xt: Th thut ny p dng cho mt lp cc bi ton gn tuyn tnh
V d 1.5. Tm tt c cc hm s f : R+ R tha f (1) =

f (xy) = f (x)f
Hint:
1. Tnh f (3)
2. Th y x3
p s: f (x) =

3
y

1
2

3
+ f (y)f
, x, y R+
x

1
2

V d 1.6. Tm tt c cc hm s f : R R tha mn iu kin:




1
= 3x, x R
f (x) + 2f
x
Hint: Th x x1
p s: f (x) = x2 x
V d 1.7. Tm tt c cc hm s f : R\{0, 1} R tha mn iu kin:


f (x) + f
Hint:
1
Th x x1
, x x1
x
1
p s: f (x) = x + 1x

Luyn tp:

x1
= 2x, x, R\{0, 1}
x

x1
x

2. Tm tt c cc hm s f : Q+ Q+ tha mn iu kin:
f (x + 1) = f (x) + 1, x Q+ v f (x3 ) = f 3 (x), x Q+
GV: Trn Minh Hin . . . . . . PTH bi dng hc sinh gii . . . . . . Trng THPT chuyn Quang Trung

www.VNMATH.com

1 PHNG PHP TH BIN

Hint:
1. Quy np f (x + n) = f (x) + n,x Q+ , n N
2. Vi

p
q

Q+ , tnh f

p
q

+ q2

3

theo hai cch.

p s: f (x) = x, x Q+
V d 1.8. (VMO 2002). Hy tm tt c cc hm s f (x) xc nh trn tp s thc R v tha mn
h thc

f (y f (x)) = f x2002 y 2001.y.f (x), x, y R.


(1)
Gii
a) Th y = f (x) vo (1) ta c

f (0) = f x2002 f (x) 2002. (f (x))2 , x R.

(2)

b) Li thay y = x2002 vo (1) th

f x2002 f (x) = f (0) 2001.x2002 .f (x), x R.


Ly (2) cng vi (3) ta c

(3)

f (x) f (x) + x2002 = 0, x R.


T y suy ra vi mi gi tr x R th ta c hoc l f (x) = 0 hoc l f (x) = x2002 . Ta s ch ra rng
tha mn yu cu bi ton th bt buc phi c ng nht
f (x) 0, x R hoc f (x) x2002 , x R.
Tht vy, v f (0) = 0 trong c hai hm s trn, nn khng mt tnh tng qut ta c th gi s tn ti
a 6= 0 sao cho f (a) = 0, v tn ti b > 0 sao cho f (b) = b2002 (v ch cn thay x = 0 vo quan h (1) ta
nhn c hm f l hm chn). Khi th x = a v y = b vo (1) ta c

f (b) = f a2002 + b .
Vy ta nhn c dy quan h sau
0 6= b2002
= f (b)
= f (b)

= f a2002 + b
"

0(mu thun v 0 6= 0)
.
2002
2002
(a2002 + b)
(mu thun v (a2002 + b)
< b2002 )

Bng cch th li quan h hm ban u ta kt lun ch c hm s f (x) 0, x R tha mn yu cu


bi ton.
V d 1.9. (Hn Quc 2003) Tm tt c cc hm s f : R R tha mn
f (x f (y)) = f (x) + xf (y) + f (f (y)) ,

x, y R.

(4)

GV: Trn Minh Hin . . . . . . PTH bi dng hc sinh gii . . . . . . Trng THPT chuyn Quang Trung

www.VNMATH.com

1 PHNG PHP TH BIN

Gii
Nhn thy hm f (x) 0 tha mn yu cu bi ton. Xt trng hp f (x) 6 0.
a) Th x = f (y) vo (4) ta c
f (0) = 2f (x) + x2 f (x) =

x2 f (0)
+
,
2
2

hay
f (f (x)) =

f 2 (x) f (0)
+
.
1
2

b) Th x = f (z), vi z l mt s thuc R th ta c
f (f (z) f (y)) = f (f (z)) + f (z)f (y) + f (f (y)) .
Vi lu l
f 2 (y) f (0)
f 2 (z) f (0)
+
v f (f (z)) =
+
,
2
2
2
2
thay vo quan h hm trn ta c
f (f (y)) =

(f (z) f (y))2
f (f (z) f (y)) =
+ f (0).
2

(5)

c) Tip theo ta chng t tp {f (x) f (y)|x, y R} = R. Do f (x) 6 0 nn tn ti mt gi tr y0 sao


cho f (y0 ) = a 6= 0. Khi t quan h (4) ta c
f (x a) = f (x) + xa + f (a) f (x a) f (x) = ax + f a.
V v phi l hm bc nht ca X nn xa + f a c tp gi tr l ton b R. Do hiu f (x a) f (x)
cng c tp gi tr l ton b R, khi x R. M
{f (x) f (y)|x, y R} {f (x a) f (x)|x R} = R,
do {f (x) f (y)|x, y R} = R. Vy t quan h (5) ta thu c
f (x) =

x2
+ f (0), x R.
2

Mt khc ta li c
x2
f (x) = + f (0), x T (f )
2
x2
nn f (0) = 0. Th li thy hm s f (x) = , x R tha mn quan h hm.
2
Kt lun: C hai hm s tha mn l f (x) =

x2
, x R hoc f (x) 0.
2

Nhn xt: Bi ton trn ly tng t bi thi IMO 1996: Tm tt c cc hm s f : R R tha


mn
f (x f (y)) = f (f (y)) + xf (y) + f (x) 1, x, y R.
p s l f (x) =

x2
+ 1, x R.
2

GV: Trn Minh Hin . . . . . . PTH bi dng hc sinh gii . . . . . . Trng THPT chuyn Quang Trung

www.VNMATH.com

1 PHNG PHP TH BIN

V d 1.10. (Iran 1999) Xc nh cc hm s f : R R tha mn

f (f (x) + y) = f x2 y + 4yf (x), x, y R.


Gii
a) Th y = x2 ta c

f f (x) + x2 = f (0) + 4x2 f (x), x R.


b) Th y = f (x) ta c

f (0) = f f (x) + x2 4 (f (x))2 , x R.


Cng hai phng trnh trn ta c

4f (x) f (x) x2 = 0, x R.
T y ta thy vi mi x R th hoc l f (x) 0 hoc l f (x) = x2 . Ta chng minh nu hm
f tha mn yu cu bi ton th f phi ng nht vi hai hm s trn. Nhn thy f (0) = 0, t
thay x = 0 ta c f (y) = f (y), y R, hay f l hm chn. Gi s tn ti a 6= 0, b 6= 0 sao cho
f (a) = 0, f (b) = b2 , khi thay x = a, y = b ta c
f (b) = f (a2 + b) f (b) = f (a2 + b).
T ta c quan h sau
0 6= b2
= f (b)
= f (b)

= f a2 + b
"

0(mu thun v 0 6= 0)
.
2
2
(a2 + b) (mu thun v (a2 + b) < b2 )

Do xy ra iu mu thun. Th li thy hm s f (x) 0 tha mn yu cu.


Nhn xt:
1. R rng bi ton VMO 2002 c tng ging bi ton ny.
2. Ngoi php th nh trn th bi ton ny ta cng c th thc hin nhng php th khc nh sau:
a) Th y =

1 2
x f (x) .
2

b) Th y = 0 c f (f (x)) = f (x2 ), sau th y = x2 f (x).


c) Th y = x f (x) v sau l y = x2 x.
V d 1.11. Tm hm s f : R R tha mn iu kin:
f (x f (y)) = 2f (x) + x + f (y), x, y R.

(6)

Gii
GV: Trn Minh Hin . . . . . . PTH bi dng hc sinh gii . . . . . . Trng THPT chuyn Quang Trung

www.VNMATH.com

1 PHNG PHP TH BIN

Nhn thy hm f (x) 0 khng tha mn yu cu. Xt f (x) 6 0.


a) Thay x bi f (y) vo (6) ta c
f (f (y)) = f (y) +

f (0)
.
2

b) Li thay x bi f (x) ta c
f (f (x) f (y)) = 2f (f (x)) + f (x) + f (y)

f (0)
= 2 f (x) +
+ f (x) + f (y)
2
= (f (x) f (y)) + f (0).
Tuy nhin vic chng minh tp {f (x) f (y)|x, y R} c tp gi tr l R cha thc hin c.
c) T y ta c
f (f (x) 2f (y)) = f ((f (x) f (y)) f (y))
= 2f (f (x) f (y)) + f (x) f (y) + f (y)
= 2 (f (x) f (y)) + 2f (0) + f (x)
= (f (x) 2f (y)) + 2f (0).
Ta s chng minh tp {f (x) 2f (y)|x, y R} bng vi R. Tht vy tn ti gi tr y0 R sao cho
f (y0 ) = a 6= 0. Khi thay y = y0 vo (6) ta c
f (x a) 2f (x) = x + a, x R.
M khi x R th x + a c tp gi tr l R. Chng t tp {f (x a) f (x)|x R} = R. M
{f (x) 2f (y)|x, y R} {f (x a) f (x)|x R} nn {f (x) 2f (y)|x, y R} = R. Do t (c)
ta kt lun f (x) = x + 2f (0), x R. Thay vo (6) ta c f (0) = 0.
Kt lun: Hm s f (x) = x, x R tha mn yu cu bi ton.
V d 1.12. (Belarus 1995) Tm tt c cc hm s f : R R tha mn
f (f (x + y)) = f (x + y) + f (x)f (y) xy, x, y R.
Gii
R rng f khc hng s.
a) y = 0 vo iu kin bi ton ta c
f (f (x)) = (1 + f (0)) f (x), x R.
b) Trong ng thc trn thay x bi x + y th
(1 + f (0)) f (x + y) = f (f (x + y)) = f (x + y) + f (x)f (y) xy,
n gin ta c
f (0).f (x + y) = f (x)f (y) xy.

(7)

GV: Trn Minh Hin . . . . . . PTH bi dng hc sinh gii . . . . . . Trng THPT chuyn Quang Trung

www.VNMATH.com

1 PHNG PHP TH BIN

c) Thay y = 1 vo (7) th
f (0)f (x + 1) = f (x)f (1) x.
d) Li thay y = 1 v x bi x + 1 vo (7) ta c
f (0).f (x) = f (x + 1).f (1) + x + 1.
Kt hp hai ng thc trn ta c

(f (0))2 f (1)f (1) f (x) = (f (0) f (1)) x + f (0).

Nu (f (0))2 f (1)f (1) = 0, th thay x = 0 vo phng trnh cui cng ta c f (0) = 0, nn theo
(7) th f (x)f (y) = xy. Khi f (x)f (1) = x, x R, iu ny dn n (f (0))2 f (1)f (1) = 1, mu
thun. Vy (f (0))2 f (1)f (1) 6= 0, suy ra f (x) l mt a thc bc nht nn c dng f (x) = ax+b. Thay
vo quan h hm ban u suy ra a = 1, b = 0. Vy hm s tha mn yu cu bi ton l f (x) = x, x R.
Nhn xt: Nu chu kh tnh ta s tnh c f (0) = 0 bng cch th cc bin x, y bi hai s 0 v 1.
V d 1.13. (VMO 2005) Hy xc nh tt c cc hm s f : R R tha mn iu kin
f (f (x y)) = f (x)f (y) f (x) + f (y) xy, x, y R.

(8)

Gii
a) Th x = y = 0 vo (8) ta c
f (f (0)) = (f (0))2 .
b) Th x = y vo (8) v s dng kt qu trn th
(f (x))2 = (f (0))2 + x2 , x R.
Suy ra (f (x))2 = (f (x))2 |f (x)| = |f (x)| , x R.
c) Th y = 0 vo (8) c
f (f (x)) = f (0)f (x) f (x) + f (0), x R ().
d) Th x = 0, y = x vo (8) c
f (f (x)) = f (0)f (x) + f (x) a, x R.
T hai ng thc trn ta c
f (0) (f (x) f (x)) + f (x) + f (x) = 2f (0), x R.

(9)

Gi s tn ti x0 6= 0 sao cho f (x0 ) = f (x0 ), th th x = x0 vo (9) ta c


f (x0 ) = f (0)
(f (x0 ))2 = (f (0))2
(f (0))2 + x20 = (f (0))2 + 02
x0 = 0 mu thun
GV: Trn Minh Hin . . . . . . PTH bi dng hc sinh gii . . . . . . Trng THPT chuyn Quang Trung

www.VNMATH.com

1 PHNG PHP TH BIN

Vy f (x) = f (x), x R, t iu ny kt hp vi (9) ta c


f (0) (f (x) 1) = 0, x R.
T y suy ra f (0) = 0, v nu ngc li th f (x) = 1, x 6= 0, tri vi iu kin f l hm l. T y ta
nhn c quan h quen thuc
(f (x))2 = x2 , x R.
Gi s tn ti x0 R sao cho f (x0 ) = x0 , khi trong (*) ta c
x0 = f (x0 ) = f (f (x0 )) = f (x0 ) = x0 ,
v l. Vy chng t f (x) = x, x R. Th li thy hm ny tha mn bi ton.
Nhn xt: Bi ton trn cho kt qu l hm chn f (x) = x. Nu vn gia nguyn v phi v
nhn c hm l f (x) = x, ta sa li d kin trong v tri nh trong v d sau
V d 1.14. Tm tt c cc hm s f : R R tha mn iu kin
f (f (x) y) = f (x) f (y) + f (x)f (y) xy, x, y R.
Gii
a) Th y = 0 ta c
f (f (x)) = f (x) f (0) + f (0).f (x), x R.

(10)

b) Th y = f (x) v s dng kt qu trn, ta c


f (0) = f (x) f (f (x)) + f (x).f (f (x)) xf (x) ()
= f (0) 2f (0).f (x) + (f (x))2 + f (0). (f (x))2 xf (x),
hay
2f (0).f (x) + (f (x))2 + f (0). (f (x))2 xf (x) = 0, x R.
c) Th x = 0 vo ng thc trn ta c
(f (0))2 (f (0))2 = 0 f (0) = 0 hoc f (0) = 1.
d) Nu f (0) = 0 th thay vo (10) ta c f (f (x)) = f (x), x R, thay kt qu ny vo trong (*) ta c
f (x) = x.
1
e) Nu f (0) = 1 thay vo (10) ta c f (f (x)) = 2f (x) 1, thay vo trong (*) ta c f (x) = x + 1.
2
Kt lun: Thay vo ta thy ch c hm s f (x) = x, x R l tha mn yu cu.
V d 1.15. (AMM,E2176). Tm tt c cc hm s f : Q Q tha mn iu kin

f (2) = 2 v f

x+y
xy

f (x) + f (y)
, x 6= y.
f (x) f (y)

Gii
GV: Trn Minh Hin . . . . . . PTH bi dng hc sinh gii . . . . . . Trng THPT chuyn Quang Trung

www.VNMATH.com

1 PHNG PHP TH BIN

Ta s chng minh f (x) = x l nghim duy nht ca bi ton da vo mt chui cc s kin sau. Trc
tin nhn thy f khng th l hm hng.
a) Tnh f (0), f (1). Thay y = 0 ta nhn c
f (1) =

f (x) + f (0)
(f (1) 1) f (x) = f (0) (1 + f (1)) , x Q.
f (x) f (0)

Suy ra f (1) = 1, f (0) = 0.


b) Hm f l hm l. Thay y = x ta c
0 = f (0) = f (x) + f (x) f (x) = f (x), x Q.
c) Thay y = cx, c 6= 1, x 6= 0 ta c


f (x) + f (cx)
1+c
1 + f (c)
=f
,
=
f (x) f (cx)
1c
1 f (c)
p
suy ra f (cx) = f (c).f (x), ly c = q, x = th ta c f
q

p
q

f (p)
f (q)

V d 1.16. Tm tt c cc hm s f : R R tha mn

f (x y)2 = (f (x))2 2xf (y) + y 2 , x, y R.


Gii
Thay x = y = 0 th (f (0)) = (f (0))2 f (0) = 0 hoc f (0) = 1.
1. Nu f (0) = 0, th thay x = y vo iu kin ban u ta c
f (0) = (f (x))2 2xf (x) + x2 = (f (x) x)2 f (x) = x, x R.
Nhn thy hm s ny tha mn.
2. Nu f (0) = 1 th li vn thay x = y = 0 ta nhn c, vi mi x R th hoc l f (x) = x + 1
hoc f (x) = x 1. Gi s tn ti gi tr a sao cho f (a) = a 1. Khi thay x = a, y = 0 ta c

f a2 = a2 4a + 1.
Nhng ta li c hoc l f (a2 ) = a2 + 1 hoc l f (a2 ) = a2 1. Do ta phi c hoc l
1
a2 4a + 1 = a2 + 1 hoc a2 4a + 1 = a2 1, tc a = 0 hoc l a = . Tuy nhin kim tra u
2
khng tha.
Vy hm s tha mn yu cu l f (x) = x, x R hoc l f (x) = x + 1, x R.
V d 1.17. (THTT T9/361) Tm tt c cc hm s f : R R tha mn iu kin

f x3 y + 2y 3 (f (x))2 + y 3 = f (x + f (y)) , x, y R.
GV: Trn Minh Hin . . . . . . PTH bi dng hc sinh gii . . . . . . Trng THPT chuyn Quang Trung

www.VNMATH.com

1 PHNG PHP TH BIN

Gii
a) Thay y = x3 ta c

f (0) + 2x3 3 (f (x))2 + x6 = f x3 + f (x) , x R.


b) Thay y = f (x) ta c

f x3 + f (x) 2f (x) 3 (f (x))2 + (f (x))2 = f (0), x R.


T hai ng thc trn ta c

2x3 3 (f (x))2 + x6 = 8 (f (x))3 , x R.


Do

0 = 4 (f (x))2 x3 3 (f (x))2 + x6

= 4 (f (x))3 4 (f (x))2 .x3 + (f (x))2 .x3 x9

= f (x) x

4 (f (x)) + x

f (x) + x

= f (x) x

x3
2f (x) +
4

15
+ x6 .
16

x3
15
Ch rng 2f (x) +
+ x6 = 0 th x = 0, f (0) = 0. Bi vy trong mi trng hp ta u c
4
16
3
f (x) = x . Th li thy hm s ny tha mn bi ton.

GV: Trn Minh Hin . . . . . . PTH bi dng hc sinh gii . . . . . . Trng THPT chuyn Quang Trung

www.VNMATH.com 2 PHNG TRNH HM CAUCHY

Phng trnh hm Cauchy

PHNG TRNH HM CSI(HM TUYN TNH) Version 5.0 updated to 24 10 2008 I.nh
ngha: Mt hm s f : R R gi l tuyn tnh nu: f (x + y) = f (x) + f (y), x, y R (Hm s tuyn
tnh cn c gi l hm Cauchy) II. Mt s tnh cht Tnh cht 1. Hm f tuyn tnh v tha mn
x 0 th`i f (x) 0, khi f l hm ng bin. (Nu vi mi x 0 f (x) 0 th hm nghch bin).
Chng minh Xt x y y x 0 f (y x) 0 Ta c f (y) = f (y x + x) = f (y x) + f (x) f (x).
Vy f l hm tng. Tnh cht 2. Hm tuyn tnh f l hm l. Chng minh Ta c f (0) = f (0 + 0) =
2f (0) f (0) = 0. T f (0) = f (x + (x)) = f (x) + f (x) = 0 f (x) = f (x), x R. Vy
f l hm l. Tnh cht 3. Hm tuyn tnh f lin tc ti x = 0 th lin tc trn ton tp s thc R.
Chng minh Xt x0 R bt k, ta c: lim [f (x) f (x0 )] = lim [f (x) + f (x0 )] = lim f (x x0 ) =
xx0

xx0

xx0

lim f (y) = f (0) = f (0) = 0 Vy hm s lin tc ti x0 R. Do x0 ly bt k trn R nn chng t hm

y0

s lin tc trn ton b R. Tnh cht 4. Hm s f tuyn tnh v ng bin trn R th lin tc trn R.
Chng minh Cho y = 0 f (x) = f (x) + f (0) f (0) = 0 Cho y = x f (2x) = 2f (x), bng quy
np ta d dng
chng minh
c: f (nx) = nf (x), n N, x R(1)
Mt khc t cng thc (1) suy

ra f (x) = nf nx hay f nx = n1 f (x), x R, n N, do : f m


x
= m
f (x), x R, m, n N
n
n
hay f (qx) = qf (x), q Q, x R n y ta c th gii quyt theo hai cch sau: Vi > 0 bt

k, chn = 1+|f (1)|+|f


, khi vi mi x R, |x| < theo tnh cht ca tp s thc th tn ti
(1)|

m
< f (x) <
m, n N sao cho |x| < n < , tc l m
<x< m
. V f l hm ng bin nn f m
n
n
n

m
m
m
m
m
f n n f (1) < f (x) < n f (1) n (1 + |f (1)| + |f (1)|) < f (x) < n (1 + |f (1)| + |f (1)|) Vy
|f (x) f (0)| = |f (x)| < m
(1 + |f (1)| + |f (1)|) < (1 + |f (1)| + |f (1)|) = . Do hm s lin
n
tc ti x = 0 nn lin tc trn R Hoc ta c th l nh sau: t f (qx) = qf (x), q Q, x R nn
f (x) = xf (1), x Q Hn na vi mi x R, tn ti hai dy hu t (un ), (vn ) Q : un < x < vn
m lim un = lim vn = x. Do hm ng bin nn f (un ) < f (x) < f (vn ) un f (1) < f (x) < vn f (1).
n

Chuyn qua gii hn ta c f (x) = f (1)x x R hay f (x) = ax nn lin tc trn R. Tnh cht 5.
Hm tuyn tnh f v lin tc trn R c biu din l f (x) = ax, (a = f (1)). Chng minh Theo cch thit
lp trong tnh cht 3 ta c f (x) = xf (1), x Q. V vi mi x R, lun tn ti dy {xn }nN Q sao
cho lim xn = x. V f lin tc nn
n

lim f (xn ) = f (x) lim xn f (1) = f (x) f (x) = ax

, vi
a = f (1)
, th li thy hm s ny tha mn yu cu bi ton. Vy f (x) = ax, x R. Tnh cht 6. Cho c > 0.
Nu hm s f tuyn tnh v tha mn iu kin |f (x)| c x [1, 1] th f (x) = ax vi |a| c Chng
minh T tnh cht 3 ta c f (qx) = qf (x), q Q, x R Gi s (xn ) l dy s thc 6= 0 tha mn
1
lim xn = 0. Vi mi gi tr ca xn ta chn mt s hu t qn tha mn: 1 qn
, n = 1, 2, ...(c
3
n
|xn |

|xn |

th t gi tr n = n0 , n0 +1, ... tha mn iu kin trn) th ta c: n


lim qn = v n
lim (xn .qn ) = 0 Vy

|f (xn )| = f


1
.q .x
qn n n

1
|f (qn xn )|,
qn

n N, do n
lim (xn .qn ) = 0 nn vi n ln th qn xn [1, 1]

nn |f (qn xn )| c, vi n ln. Do |f (xn )|

1
c
qn

Do lim f (xn ) = 0 = f (0) nn hm f lin tc


n

ti 0, t lin tc trn ton b R do c biu din f (x) = ax. T iu kin bi ton ta c hm cn


tm l f (x) = ax vi |a| c Tnh cht 7. Nu hm s f tuyn tnh v tha mn iu kin tn ti hng
s M > 0 sao cho f (x) M x [0, 1] thf (x) = ax Chng minh T f (qx) = qf (x) q Q, x R
GV: Trn Minh Hin . . . . . . PTH bi dng hc sinh gii . . . . . . Trng THPT chuyn Quang Trung

www.VNMATH.com 2 PHNG TRNH HM CAUCHY


hay f (x) = ax x Q T iu kin bi ton ta c: f (1) f (x) = f (1 x) M x [0, 1], Suy ra
f (1) M f (x) M x [0, 1] Vy tn ti hng s N > 0 m |f (x)| N x [0, 1] |f (x)|
N x [1, 1](do f (x) = f (x)), n y ta c th l tip theo nh tnh cht
6. y tacth
chng



x
x
+
minh khc nh sau: Vi mi x R, khi vi r Q sao cho |x| < r th r 1, do f r N . V
1
r

Q nn f

x

r

= 1r |f (x)| N |f (x)| r.N Cho r |x| th |f (x)| N |x|. Suy ra lim f (x) = 0
x0

hay f lin tc ti 0 nn lin tc trn ton b R. Do f (x) = ax Nhn xt 1. Cho tp A = R, [0, )


hay (0, ). Nu f : A R tha mn f (x + y) = f (x) + f (y) v f (xy) = f (x)f (y), x, y A, th
hoc l f (x) = 0, x A hoc l f (x) = x, x A Chng minh Theo tnh cht ca hm cng tnh th
f (x) = f (1).x, x Q. Nu f (1) = 0 th f (x) = f (x.1) = f (x).f (1) = 0, x A. Nu f (1) 6= 0 do
T

f (1) = f (1)f (1) f (1) = 1 f (x) = x, x A Q Nu y 0 th f (y) = f ( y)f ( y) = f 2 ( y) 0


v do f (x + y) = f (x) + f (y) f (x), hay chng t f l hm tng. By gi vi mi x A\Q, theo
tnh tr mt ca tp s thc, tn ti hai dy pn , qn Q sao cho pn < x < qn ; pn % x v qn & x,
khi n . Do f l hm tng, ta c: pn = f (pn ) f (x) f (qn ) = qn Chuyn qua gii hn ta c
f (x) = x, x A III. Cc h qu trc tip ca hm Cauchy T quan h cho hm f lin tc tha mn
iu kin f (x + y) = f (x) + f (y) ta c biu din ca hm l f (x) = ax. Nu ta t vo quan h hm trn
qua php logarit Nepe tc l: ln f (x+y) = ln f (x)+ln f (y) = ln(f (x).f (y)), suy ra f (x+y) = f (x).f (y).
Vy nu f (x) > 0 vi mi x R th quan h hm f (x + y) = f (x).f (y) d dng chuyn v quan h hm
Cauchy qua php logarit. Tuy nhin t quan h hm d dng thy c bi ton vn gii c vi
min xc nh trn R.
"
f (x) 0
H qu 1. Cc hm s lin tc trn R tha mn iu kin: f (x+y) = f (x).f (y) (1) l:
f (x) = ax (a > 0)
Chng minh Nhn thy hm ng nht f (x) 0 tha mn quan h . Xt hm khng ng nht 0,
khi tn ti x0 :f (x0 ) 6= 0 th: f (x0 ) = f ((x0 x) + x) = f (x0 x)f
(x) 6= 0 f (x) 6= 0 x R V

x
x
2 x
cng tha iu kin lun dng, tht vy: f (x) = f 2 + 2 = f 2 > 0 x R Do n y ta
ch cn t ln f (x) = g(x) th ta c quan h: g(x + y) = g(x) + g(y) Vy g(x) = bx, b R ty . Vy
f (x) = ebx = ax (a > 0). Vy hai hm tha mn quan h l:
By gi li t hm Cauchy nu ta nng ly tha ca bin ln t x thnh ex ta c quan h l
x+y
f (e ) = f (ex ) + f (ey ) g(x + y) = g(x) + g(y) vi g(x) = f (ex ) v hm g thu c li chnh l hm
Cauchy. Mt khc t f (ex+y ) = f (ex ) + f (ey ) f (ex .ey ) = f (ex ) + f (ey ), by gi thay ngc tr li ex
bi x th ta c quan h mi l f (xy) = f (x) + f (y). Quan h ny vi quan h Cauchy tng tc vi
nhau bi vic nng ly tha ca bin. Tuy nhin vic nng ly tha ca bin li c yu cu bin phi
dng. Nu c mt bin bng 0 th bi ton tr nn d dng vi kt qu l f (x) 0, nu c hai bin
cng dng th bi ton chuyn v phng trnh hm Cauchy qua php nng bin ln ly tha. Nu c
hai s cng m th tch xy l s dng nn li quy v trng hp hai bin cng dng.
H qu 2. Cc hm s f (x)lin tc trn R\{0} tha mn iu kin:f (xy) = f (x) + f (y) x, y R
(2)l: f (x) = b ln |x| x R\{0}, b R Chng minh Nu x = y = 1 th t (3) ta c f (1) = 0. Li
cho x = y = 1 ta c f (1) = 0. By gi cho y = 1 th ta c f (x) = f (x) x R. Do f l
hm chn. a) Xt x, y R+ , t x = eu , y = ev , f (eu ) = g(u) ta c g(u + v) = g(u) + g(v) u, v R
g(t) = bt f (x) = a ln x x R+ , a R b) Nu x, y R th xy R+ nn vi y = x ta c:
f (x) = 21 f (x2 ) = 12 b ln(x2 ) = b ln |x| x R , b R
Li tip tc t quan h hm f (x + y) = f (x).f (y) ta li nng bin theo ly tha ca e th c dng
f (ex+y ) = f (ex )f (ey ) f (ex .ey ) = f (ex )f (ey ) v ta c quan h hm: g(xy) = g(x)g(y) Hin nhin
bi ton c ngay li gii nu min xc nh cha s 0. Do ta t vn nh sau:
H qu 3. Cc hm f (x) lin tc trn R\{0} tha mn iu kin:f (xy) = f (x)f (y), x, y R\{0}l:

GV: Trn Minh Hin . . . . . . PTH bi dng hc sinh gii . . . . . . Trng THPT chuyn Quang Trung

www.VNMATH.com 2 PHNG TRNH HM CAUCHY


8
<

f (x) = 0 f (x) = |x| f (x) =

x , x R+
|x| , x R

Chng minh Thay y = 1 f (x)(1 f (1)) =

0, x R\{0} (1) Nu f (1) 6= 1 th t (1) suy ra f (x) 0, x R\{0} Xt f (1) = 1, khi


1 = f (1) = f x. x1 = f (x)f x1 , x R\{0}. Vy f (x) 6= 0, x R\{0}. a) Xt x, y R+ , t
x = eu , y = ev va g(t) = f (et ). Khi ta c: g(u + v) = g(u)g(v), u, v R Vyg(t) = at t R(a >
0 tuy y`
u) v do : f (x) = f (eu ) = g(u) = au = aln x = xln a = x , x R+ trong = ln a b)
By gi ta xt trng hp
x 6= 0, y 6= 0 bt k th cho v x = y = t ta nhn c f 2 (t) = f (t2 ) =
"
f (t) = f (t) = tc (hay 0)
Vy trong trng hp tng qut ta c cc nghim
f (t)f (t) = f 2 (t)
f (t) = f8(t) = tc
<

l: a) f (x) = 0 b) f (x) = |x| f (x) = :

x , x R+
|x| , x R

T quan h hm Cauchy f (x + y) = f (x) + f (y) ta thc hin v tri theo trung bnh cng v tri
theo bin v trung bnh cng v phi theo hm s th ta nhn c:

(y)
= f (x)+f
(4) l:f (x) = ax+b
H qu 4(Hm Jensen). Cc hm f (x) lin tc trn R tha mn f x+y
2
2


f (x)+f (0)
f (x+y)+f (0)
f (x)+f (y)
x+y
x
. Vy:
=f 2 =
f (x + y) + f (0) =
Chng minh Cho y = 0 f 2 =
2
2
2
f (x) + f (y) t g(x) = f (x) f (0) th ta c g(x + y) = g(x) + g(y) hay g(x) = ax f (x) = ax + b
Li trong quan h hm Jensen
ta thc hin logarit Nepe ni ti ca bin(d nhin trong trng

(ln y)
(ln y)
ln x+ln y
= f (ln x)+f
f (ln xy) = f (ln x)+f
. T vn ny t
hp cc bin dng, ta c:f
2
2
2
+
ngc
li
ta c h qu sau: H qu 5. Cc hm f (x) xc nh v lin tc trn R tha mn iu kin:

(y)
f
xy = f (x)+f
x, y R+ (5) l f (x) = a ln x + b iu kin x, y R+ l cho hm s lun c
2
xc nh.
Chng
minh t x = eu , y = ev , g(u) = f (eu ). Khi g(u) lin tc trn R v tha mn iu

= g(u)+g(v)
kin: g u+v
u, v R Suy ra g(u) = au + b f (x) = a ln x + b, x R+ .
2
2

(y)
= f (x)+f
nu ta vit c vo di dng ca biu din logarit
Cng li t quan h hm f x+y
2

ln f (x)+ln f (y)
x+y
x+y
x+y
=

ln
f
=
ln
f
(x)f
(y)

f
=
f (x)f (y) Tc l ta
tc l: ln f 2
2
2
2

x+y
c quan h hm: f 2 = f (x)f (y). Vy ta c: H qu 6. Hm s f : R R lin tc tha
2

f (x) 0
4
=
f x+y
f
(x)f
(y)
(6)
l:
2
f (x) = eax+b (a, b R)

Chng minh
T iu kin bi ton cho x = y f (x) = f 2 (x) 0. Nu tn ti x0 : f (x0 ) = 0 th:

x0 +y
f 2 = f (x0 )f (y) = 0 y R tc l f (x) 0 Nu f (x) > 0 th thc hin logarit Nepe hai v a
v hm Jensen ta c:f (x) = eax+b , a, b ty thuc R. T ta c iu phi chng minh.
Li t quan h hm trong h qu 5, thc hin php ton nghch o hm s(gi s thc hin
c) ta c:

1
f ( xy)

1
1
+ f (y)
f (x)

, bng cch t g(x) =

hm f (x) xc nh v lin tc trn R

1
f (x)

ta nhn c h qu sau: H qu 7. Cc

tha mn iu kin:f ( xy) = 1 +2 1 x, y R+ (7) l


f (x)

f (y)

hm hng f (x) = b R\{0} Chng minh T gi thit bi ton suy ra f (x) 6= 0 x R+ . Ta


1
+ 1

1
c f (1xy) = f (x) 2 f (y) g( xy) = g(x)+g(y)
x, y R+ vi g(x) = f (x)
Theo h qu 5 th g(x) =
2
a ln x + b f (x) = a ln1x+b . f (x) lin tc trn R+ th: a ln x + b 6= 0, x R+ nn a = 0, b 6= 0. Vy
f (x) = b R\{0}(pcm).
1

f(

x+y
2

1
1
+ f (y)
f (x)

=
2
)
f (x)+f (y)
(x)f (y)
hay f x+y
= f2f(x)+f
Tuy nhin m bo cho php nghch o hm lun thc hin c
2f (x)f (y)
2
(y)
th ta ch cn gii hn gi tr hm trong R+ . Do ta nhn c kt qu: H qu 8. Hm s f : R R+
T quan h hm Jensen nu ta thc hin nghch o(vi hm s) th ta c:

GV: Trn Minh Hin . . . . . . PTH bi dng hc sinh gii . . . . . . Trng THPT chuyn Quang Trung

www.VNMATH.com 2 PHNG TRNH HM CAUCHY

(x)f (y)
= f2f(x)+f
(8) l f (x) = 1b , b > 0
lin tc tha mn f x+y
2
(y)
1
Chng minh Ch cn t g(x) = f (x)
, ta nhn c quan h hm Jensen theo hm g(x) nng(x) =
1
cx + d. Do f (x) = cx+d . Tuy nhin hm s ny cn phi tha mn iu kin f (x) R+ nn:
1
> 0, x R c = 0, b > 0, vy hm thu c l f (x) = 1b , b > 0 ty .
cx+d
Li vn trong quan h hm Jensen nu ta thc hin php bnh phng vo hm s th ta nhn ngay
c h qu sau:
q

[f (x)]2 +[f (y)]2


(9) l f (x) = c vi c 0. Chng
H qu 9. Hm s f (x)lin tc trn R tha f x+y
=
2
2

(y)]
minh T quan h hm s suy ra f (x) 0, x R. Ta c: f x+y
= [f (x)] +[f
. t g(x) = [f (x)]2
2
2

th ta nhn c
quan h hm Jensen cho hm g(x)nn g(x) = ax + b. Do f (x) = ax + b. M theo
iu kin th ax + b 0, x R a = 0, b 0 Ta c hm f (x) = b, b 0.
T quan
h
hm
trong h qu 6, nu ta thc
hin php nng ly tha ln c s e(i vi bin) th



x y
x+y
x
y
ta c: f e 2 = f (e )f (e ) f ( e .e ) = f (ex )f (ey ) Thay ngc li bin d ng bnh thng ta
nhn c kt qu:

H qu 10. Hm s f (x) xc nh v lin tc trn R+ tha f ( xy) = f (x)f (y), x, y R+ (10) l:


"

f (x) 0
Chng minh t x = eu , y = ev , f (eu ) = g(u) th ta nhn c: g u+v
=
a
2
f (x) = c.x , a R, c > 0
2
"

f (x) 0
g(u) 0
g(u)g(v), theo h qu 6 th:
. Vy 4
. Trong quan
au
g(u) = e + b
f (x) = ea ln x+b = c.xa , c > 0, a R
2
2 (y)

h hm ca h qu 5, nu ta thc hin theo quan h hm bnh phng, tc l f 2 ( xy) = f (x)+f


,
2
thc hin cn bc hai
hai v ta c h qu 11. H qu 11. Hm s f (x) xc nh v lin tc trn
q

f 2 (x)+f 2 (y)
+
R tha f ( xy) =
, x, y R+ (11) l f (x) c, c 0 Chng minh T gi thit ca
2
hm
d
thy f (x) 0, x R+ . t x = eu , y = ev , [f (eu )]2 = g(u). Khi g(u) 0, v ta c:

g u+v
= g(u)+g(v)
, u, v R Vy g(u) = au + b. g(u) 0, u Rth a = 0, b 0. Do
2
2
f (x) c, c 0.


f (x)+f (y)
1
=
th ta nhn c
Li t quan h hm Jensen f x+y
,
ta
xt
php
gn
hm
f
(x)
=
g
2
2
x




1
1
1
1
g
g
+g
+g
( ) ( )
( ) ( )
1
2
quan h hm s: g (x+y)/2
= x 2 y g x+y
= x 2 y , thay ngc tr li bin bnh thng
ta c: H qu 12. Hm s f (x) lin tc trn R\{0} tha mn

1
x

2
+

f (x) + f (y)
, x, y, x + y 6= 0
2

1
y

(12) l hm s f (x) = xa + b; a, b R ty . Gii Vi cch thit lp nh trn th ta c g(x) = ax + b,


(y)
vi g(x) = f x1 , khi th f (x) = xa + b; a, b R. Li t quan h hm Jensen f x+y
= f (x)+f
, ta
2
2
1
xt php gn hm f (x) = g 1 th ta nhn c quan h hm:
(x)

1
1
x+y
2

1
g ( x1 )

+
2

1
g ( y1 )

1
x

2g

+g

1
x

1
y

1
y

2
g
x+y

2g
g

1
x

1
x

+g

1
y

1
y

1
g ( x1 )

2
+

1
g ( y1 )

Thay ngc li bin ta c: H qu 13. Hm s f (x) xc nh lin tc trn R\{0} tha f 1 +2 1 =


x
y
2
x
f (x) = , a 6= 0
6
a
2
6
. Bng cch thc hin cc php ton khai cn, nng ly tha, logarit
1
1 (13) l 4
+
1
f (x)
f (y)
f (x) = , b 6= 0
b
GV: Trn Minh Hin . . . . . . PTH bi dng hc sinh gii . . . . . . Trng THPT chuyn Quang Trung

www.VNMATH.com 2 PHNG TRNH HM CAUCHY


Nepe nh trong cc phn trc ta thu c cc kt qu tng t sau: H2qu 14. Hm s f (x) xc nh

f (x) 0
lin tc trn R\{0} tha f 1 +2 1 = f (x)f (y), x, y, x + y 6= 0(14) l: 4
H qu
a
x
y
f (x) = e x +b , a, b R
q

[f (x)]2 +[f (y)]2


, x, y, x
2

+ y 6= 0 (15) l:


x2 +y 2
+
=
f (x) c, c 0 ty . H qu 16. Cc hm f (x) 0 xc nh lin tc trn R tha f
2
q

2
2
[f (x)] +[f (y)]
, x, y R+ (16) l: f (x) = ax2 + b vi a, b 0 ty . H qu 17. Cc hm s f (x) xc
2


x2 +y 2
(y)
nh, lin tc trn R v tha f
= f (x)+f
, x, y R (17) l: f (x) = ax2 + b; a, b R H
2
2



x2 +y 2
= f (x)f (y), x, y R (18) l:
qu 18. Cc hm s f (x) xc nh, lin tc trn R tha f
2
2


f (x) 0
x2 +y 2
4
=
H
qu
19.
Cc
hm
s
f
(x)
xc
nh,
lin
tc
trn
R
tha
f
2
2
f (x) = eax +b ; a, b R
2
, x, y R (19) l: f (x) = ax21+b vi ab 0, b 6= 0 ty .
1
+ 1
15. Hm s f (x) xc nh lin tc trn R\{0} tha f

f (x)

1
+ y1
x

f (y)

IV. Cc bi tp vn dng Bi ton 1. Tm tt c cc hm f (x) lin tc trn R tha: f (x + y) =


f (x)+f (y)+f (x)f (y) Gii: T bi ton ta c: f (x+y)+1 = (f (x)+1)(f (y)+1) nn t g(x) = f (x)+1
th ta c g(x+y) = g(x).g(y) g(x) = ax vy f (x) = ax 1. Bi ton 2. Tm tt c cc hm s f (x) lin
tc trn R tha mn iu kin:f (x)+f (y)f (x+y) = xy, x, y R Gii Ta c th vit li phng trnh
1
f (x) + f (y) f (x + y) = [(x + y)2 (x2 + y 2 )]
2
t g(x) = f (x) + 21 x2 th ta c
hm di dng:
1 2
1 2
1
2
f (x) + x + f (y) + y = f (x + y) + (x + y)
2
2
2
g(x) l hm lin tc trn R tha mn iu kin: g(x) + g(y) = g(x + y) Vy g(x) = ax, x R, a l mt
hng s thc, nn f (x) = 21 x2 + ax. Th li thy hm ny tha mn yu cu bi ton. Bi ton 3. Cho
a R, tm tt c cc hm lin tc f : R R sao cho: f (x y) = f (x) f (y) + axy, x, y R Gii Cho
x = 1, y = 0 f (1) = f (1) f (0) nn f (0) = 0. Li cho x = y = 1 f (0) = f (1) f (1) + a a = 0.
Vy vi a 6= 0 th khng tn ti hm s. Ta vit li quan h hm f (x y) = f (x) f (y), x, y R
T y ta c: f (x) = f (x + y y) = f (x + y) f (y) f (x + y) = f (x) + f (y), x, y R Vy
f (x) = ax,
x R Bi ton 4. Tm tt c cc hm s f (x) xc nh lin tc trn R+ tha mn iu

kin:f xy = f (x) f (y) x, y R+ Gii t xy = t x = ty thay vo ta c: f (t) = f (ty) f (y)
f (ty) = f (t) + f (y). Vy f (x) = a ln x x R+ , a R.
Bi ton 5. Cho a, b R\{0}, tm cc hm f (x) xc nh lin tc trn R v tha mn iu kin:
f (ax + by) = af (x) + bf (y) x, y R(1) Gii Cho x = y = 0 vo (1) ta c: f (0)(a + b 1) = 0
Nu a + b 6= 1 th f (0) = 0. Vy iu kin Cauchy c tha mn, nn khi th f (ax) = af (x)
v f (bx) = bf (x), v ta c quan h f (ax + by) = f (ax) + f (by), x, y R. Vy f (x) = x. Nu
a + b = 1 th nhn gi tr ty , vy ta phi t mt hm mi c quan h Cauchy l g(x) =
f (x) f (0) th g(0) = 0 v tng
t nh phn trnh by trn ta c f (x) = cx + d Vy: f (ax + by) =
"
a + b = 1 f (x) = cx, c R
af (x) + bf (y) x, y R l:
Nhn xt: Vi cch lm tng t
a + b = 1 f (x) = cx + d, c, d R
cho quan h f (ax + by) = af (x) + bf (y) Bi ton 6. Xc nh cc hm s f lin tc trn R tha
mn iu kin:f (2x y) = 2f (x) f (y), x, y R Gii t g(x) = f (x) f (0) th g(0) = 0, t
phng trnh trn ta thu c: g(2x y) = 2g(x) g(y), x, y R Cho y = 0 g(2x) = 2g(x)
v cho x = 0 g(y) = g(y). Thay vo trn ta c: g(2x y) = g(2x) g(y), x, y R Vy
y
g(x+y) = g 2. x2 1. 1
= g(x)g(y) = g(x)+g(y), x, y R. Do : g(x) = ax, x R, a l s thc
GV: Trn Minh Hin . . . . . . PTH bi dng hc sinh gii . . . . . . Trng THPT chuyn Quang Trung

www.VNMATH.com 2 PHNG TRNH HM CAUCHY


ty . Vy f (x) = ax+b, th li thy hm ny tha mn yu cu bi ton. Bi ton 8( ngh IMO 1979).
Chng minh rng mi hm f : R R tha mn iu kin: f (xy+x+y) = f (xy)+f (x)+f (y), x, y R
khi v ch khi f (x + y) = f (x) + f (y), x, y R Gii D thy nu f tuyn tnh th f tha mn h
thc u tin. Gi s f (xy + x + y) = f (xy) + f (x) + f (y), x, y R t y = u + v + uv ta c:
f (x + u + v + xu + xv + uv + xuv) = f (x) + f (u + v + uv) + f (xu + xv + xuv) Hon i vai tr ca x v u
ta c: f (u + x + v + ux + uv + xv + uxv) = f (u) + f (x + v + xv) + f (ux + uv + uxv) So snh hai ng
thc trn ta c: f (x) + f (u + v + uv) + f (xu + xv + xuv)= f (u) + f (x + v + xv) + f (ux + uv + uxv)
Hay f (uv) + f (xu + xv + xuv) = f (xv) + f (xu + uv + xuv) Ly x = 1 ta c f (u) + 2f (uv) =
f (u + 2uv), theo v d 4 ta c iu phi chng minh. Bi ton 9. Tm tt c cc hm s f (x) lin
tc trn R tha mn iu kin:f (x)f (y) f (x + y) = sin x. sin y, x, y R Gii Thay y = 0 ta c
f (x)[f (0) 1] = 0 f (0) = 1, v d dng nhn thy f (x) 0, x R khng l nghim ca phng
2
trnh. Thay y = x ta nhn c: f (x)f (x) f (0) = sin
x, x
R f (x)f (x)
= 1 sin2 x =

2
cos x, x R(1). Thay x = 2 vo (1) ta c nn: f 2 .f 2 = 0 Hoc f 2 = 0 thay vo

hm ta c: f x + 2 = sin x f x + 2 = sin x f (x) = sin x 2 = cos x, x R

Hoc f 2 = 0 thay vo hm ta c: f x 2 = sin x f (x) = sin x + 2 = cos x, x R


D dng kim tra li thy f (x) = cos x l hm tha mn yu cu bi ton. Bi ton 10. Tm tt c
cc hm s f : R R tha mn f (x + y xy) + f (xy) = f (x) + f (y) (1) vi mi x, y R. Gii Ta
chng minh nu f l hm s tha mn iu kin bi ton th hm s F (x) = f (x + 1) f (x) s tha
mn iu kin hm Cauchy F (u + v) = F (u) + F (v) vi mi (u, v) = {(u, v) : u + v > 0hoc
u = v = 0 hoc u + v 4} Tht vy, gi s f l hm s tha mn iu kin (1). Ta nh ngha
hm s f (x, y) bi: f (x, y) = f (x) + f (y) f (xy) D thy rng hm f tha mn phng trnh hm:
f (xy, z) +f (x, y) = f (x, yz) + f (y, z)(1) Mt khc
ta c f (x, y) = f (x + y xy)(2) Thay (2) vo (1)

ta c: f xy + y1 x + f (x + y xy) = f (1) + f y + y1 1 , vi mi x, y 6= 0 t xy + y1 x = u + 1
v x + y xy = v + 1(3) ta nhn c: f (u + 1) + f (v + 1) = f (1) + f (u + v + 1), vi mi u, v tha
mn iu kin trn. Bng vic cng hai ng thc ca (3) ta c y + y1 = u + v + 2, c nghim y 6= 0
ch trong trng hp D = {(u + v + 2)2 4 = (u + v)(u + v + 4) 0}. iu kin ny xy ra khi v
ch khi hoc l u + v > 0 hoc u + v = 0 hoc u + v + 4 0. Bng vic kim tra iu kin ta thy bi
ton c tha. Nu f l mt nghim ca bi ton th f phi c dng f (x) = F (x 1) + f (1)(1) vi
mi x, trong F tha mn phng trnh hm Cauchy F (x + y) = F (x) + F (y) vi mi x, y. Chng
minh Theo chng minh trn, th f c dng vi F tha mn phng trnh Cauchy vi mi (u, v) .
Ta s chng minh rng F tha mn phng trnh Cauchy vi mi (u, v) bt k. Gi s , khi tn ti
mt s thc sao cho cc im (x, u), (x + u, v), (x, u + v) nm trong vi vic xc nh x l: c nh
(u, v) th t cc bt ng thc x + u > 0, x + u + v > 0 ta tm c iu kin ca x. Nhng khi :
F (u) = F (x + u) F (x)
F (v) = F (x + u + v) F (x + u) Suy ra t cc phng trnh ny ta c F (u) + F (v) = F (u + v). V bi
F (u + v) = F (x + u + v) F (x)
ton c chng minh.
Bi ton 14(VMO 1992 bng B). Cho hm s f : R R tha mn f (x + 2xy) = f (x) + 2f (xy),
x, y R. Bit f (1991) = a, hy tnh f (1992) Gii Thay x =
0 ta c f (0) = 0. Thay y = 1 ta

x
1
f (x) = 2f 2 . Xt x 6= 0 v s thc t bt k, t
nhn c f (x) = f (x). Thay y = 2 ta c

t
t
y = 2x ta nhn c: f (x + t) = f (x) + 2f 2 = f (x) + f (t) Vy f l hm Cauchy nn f (x) = kx,
a
vi k l hng s no . T f (1991) = a k.1991 = a k = 1991
. Do f (1992) = 1992
a Bi ton
1991
15. Tm tt c cc hm s f (x) xc nh trn (0, +), c o hm ti x = 1 v tha mn iu kin

f (xy) = xf (y) + yf (x), x, y R+ Gii Xt cc hm s sau g(x) = f(x)


. T gi thit ca bi ton
x

+
ta c: xy.g(xy) = xy.g(x) + xy.g(y) g(xy) = g(x) + g(y), x, y R Vy g(x) = loga x, x > 0.
GV: Trn Minh Hin . . . . . . PTH bi dng hc sinh gii . . . . . . Trng THPT chuyn Quang Trung

www.VNMATH.com 2 PHNG TRNH HM CAUCHY

T ta c kt qu hm s f (x) = k. x.loga x vi k R. Li t (1) nu ta t z = x + y th y = z x


v quan h (1) tr thnh f (z) = f (x).f (z x), nu vi gi thit f (x) 6= 0 x R th ta c th vit li
(z)
, v ta xut c bi ton sau y: Bi ton 18. Xc nh cc hm s f (x)
nh sau: f (z x) = ff (x)
8
f (x)
>
< f (x y) =
, x, y R
f (y)
(2) V gi thit l f (x) 6= 0 x R
lin tc trn R tha mn iu kin: >
:
f (x) 6= 0 x R
x
nn ch c hm s f (x) = a (a > 0) tha mn yu cu bi ton.
To be continued
.

GV: Trn Minh Hin . . . . . . PTH bi dng hc sinh gii . . . . . . Trng THPT chuyn Quang Trung

www.VNMATH.com

3 PHNG PHP QUY NP

Phng php quy np

Phng php ny yu cu ta trc


ht tnh f (0), f (1) ri da vo tnh f (n) vi n N. Sau

tnh f (n) vi n Z. Tnh tip f n1 , t suy ra biu thc ca f (r) vi r Q. Phng php ny
thng s dng khi cn tm hm s xc nh trn N, Z, Q.
V d 3.1. Tm tt c cc hm s f : Q Q tha mn iu kin:
f (1) = 2,

f (xy) = f (x)f (y) f (x + y) + 1, x, y Q.

(11)

Gii
Cho y = 1 v s dng gi thit f (1) = 2 ta c
f (x + 1) = f (x) + 1, x Q.

(12)

Bng phng php quy np ta chng minh c


f (x + m) = f (x) + m, x Q, m N.

(13)

Tip theo ta s ln lt chng minh:


a) f (n) = n + 1, n N. Tht vy trong (12) cho x = 0 ta tm c f (0) = 1. Gi s ta c
f (k) = k + 1 th
f (k + 1) = f (k) + 1 = k + 1 + 1 = k + 2.
b) Tip theo ta chng minh f (m) = m+1, m Z. Tht vy, trong (12) cho x = 1 ta c f (1) = 0.
Trong (11) cho y = 1 th ta c
f (x) = f (x 1) + 1, x Q.
Khi vi m Z, m < 0 th t n = m, khi n N nn s dng kt qu trn v phn (a) ta
c
f (m) = f (n) = f (n 1) + 1 = n + 1 = m + 1.


1
, n N+ , bng cch trong
c) Tip theo ta chng minh f (x) = x + 1, x Q. Trc tin ta tnh f
n
1
(11) cho x = n, y = ta c
n


1
1
2 = (n + 1)f
f n+
+ 1.
n
n
Li theo (13) th

1
1
f n+
=f
+n
n
n
thay vo phng trnh trn ta c


1
n+1
1
f
=
= + 1.
n
n
n
GV: Trn Minh Hin . . . . . . PTH bi dng hc sinh gii . . . . . . Trng THPT chuyn Quang Trung

www.VNMATH.com

3 PHNG PHP QUY NP

T y th vi x Q th x lun c biu din di dng x =

m
, m Z, n N+ , do
n

m
n 

1
= f m.
n

f (x) = f

1
1
f m+
+1
n
n


 
1
1
= (m + 1).
+1 f
m+1
n
n


1
1
= (m + 1)
+1 1m+1
n
n
m
=
+1=x+1
n
= f (m).f

Th li thy hm s f (x) = x + 1, x Q tha mn yu cu bi ton.


Nhn xt: Bi ton trn kt qu khng thay i nu ta lm trn tp R v khng cn cho trc
f (1). Vic cho trc f (1) gip qu trnh quy np thun li hn. T li gii trn ch cn s l trn tp
s v t. Tham kho thm v bi ny trong bi 8.11.
V d 3.2. Tm tt c cc hm s lin tc f : R R tha mn
f (x + y) + f (x y) = 2 (f (x) + f (y)) , x, y R.
Gii
a) f (0) = 0, tht vy ch cn thay x = y = 0 ta c c kt qu.
b) f l hm chn. i vai tr gia x, y trong iu kin ta c
f (x + y) + f (y x) = 2 (f (x) + f (y)) , x, y R.
V nh vy th f (x y) = f (y x), x, y R. Do f l hm chn nn ta ch cn lm vic trn R+ .
c) f (nx) = n2 f (x), n N, x R+ . Tht vy, cho x = y ta c
f (2x) = 4f (x), x R+ .
Gi s ta c f (nx) = n2 f (x), n N, x R+ . Khi thay y = nx ta c
f ((n + 1)x) + f ((n 1)x) = 2 (f (x) + f (nx)) ,
hay

f ((n + 1)x) = 2 f (x) + n2 f (x) (n 1)2 f (x) = (n + 1)2 f (x).


d) f (qx) = q 2 f (x), x R+ , q Q+ . Tht vy t (c) th

x
1
1
= 2 f (x), n N, x R+ .
f (x) = 2 f (nx) f
n
n
n

Vi q Q+ th q =

m
vi m, n N, n 6= 0 nn
n


x
m2
x
2
f (qx) = f m.
=m f
= 2 f (x) = q 2 f (x).
n
n
n

GV: Trn Minh Hin . . . . . . PTH bi dng hc sinh gii . . . . . . Trng THPT chuyn Quang Trung

www.VNMATH.com

3 PHNG PHP QUY NP

e) Do f lin tc trn R+ nn f (x) = ax2 , x R+ (vi a = f (1)).


Th li thy hm s f (x) = ax2 , x R tha mn yu cu bi ton.

Nhn xt: Quan h bi ton trn chnh l ng thc hnh bnh hnh quen thuc. l nu
u ,
v
l hai vector th ta c


2
2
2
2

|
u +
v | + |
u
v | = 2 |
u | + |
v|
Bn cht ca li gii l chng minh nu hm f lin tc v tha mn hng ng thc hnh bnh hnh
th bt buc phi c dng f (x) = f (1)x2 . Cng cn lu l iu kin lin tc c th thay bng iu
kin n iu ca hm s.
V d 3.3. Tm tt c cc hm s f : [0, ) R sao cho f n iu v tha mn iu kin

(f (x) + f (y))2 = f x2 y 2 + f (2xy), x y 0.


Gii
1
Cho x = y = 0 ta c f (0) = 0 hoc f (0) = .
2
1
1
1
a) Trng hp f (0) = , th thay x = 1, y = 0 ta li c f (1) = hoc f (1) = .
2
2
2
1
1
(i) Nu f (1) = th thay x = y = 1 ta c f (2) = . Khi ta thy f (0) > f (1), f (1) < f (2),
2
2
mu thun vi tnh cht n iu ca hm s.
1
(ii) Vy f (1) = . Khi thay x = y ta c
2

1
4 (f (x))2 = f 2x2 + .
2
Xt dy s x1 = 1, xn+1 = 2x2n , thay vo quan h trn ta c
1
4 (f (xn ))2 = f (xn+1 ) + .
2
1
vi mi n Z+ . V xn v f n iu nn suy ra
Bng quy np ta c f (xn ) =
2
1
f (x) = vi mi x 0.
2
b) Trng hp f (0) = 0. Khi thay y = 0 ta c

f x2 = (f (x))2 , x 0 f (x) 0, x 0.
Ngoi ra thay x = y ta c 4 (f (x))2 = f (2x2 ). Kt hp vi ng thc trn ta c
4f (x) = f (2x), x 0.
Trong phng trnh hm ban u, t x = u + v, y = u v th ta c

[f (u + v) f (u v)]2 = f (4uv) + f 2(u2 v 2 )

= 4 f (2uv) + f (u2 v 2 )
= 4 (f (u) + f (v))2 .

GV: Trn Minh Hin . . . . . . PTH bi dng hc sinh gii . . . . . . Trng THPT chuyn Quang Trung

www.VNMATH.com

3 PHNG PHP QUY NP

T y ly cn bc hai ta c
f (u + v) + f (u v) = 2 (f (u) + f (v)) , u v 0.
Phng trnh hm ny c nghim l f (x) = f (1)x2 , x 0. Ngoi ra d dng tnh c f (1) = 0
hoc f (1) = 1.
Kt lun: Cc hm s tha mn l f (x) 0, f (x)

1
v f (x) = x2 , x 0.
2
2

Nhn xt: Bi ton trn xut pht t mt hng ng thc quen thuc l (x2 + y 2 ) = (x2 y 2 ) +
(2xy)2 . V im mu cht ca bi ton l tnh cht f (x2 ) = (f (x))2 , suy ra f (x) 0 khi x 0.
V d 3.4. (China 1996) Cho hm s f : R R tha mn iu kin:
f (x3 + y 3 ) = (x + y)(f 2 (x) f (x)f (y) + f 2 (y)), x, y R.
Chng minh rng f (1996x) = 1996f (x), x R.
Gii
a) Tnh f (0) v thit lp cho f (x).
Cho x = y = 0 ta c f (0) = 0. Cho y = 0 ta c
f (x3 ) = xf 2 (x).
Nhn xt: f (x) v x lun cng du. T y ta c
1

f (x) = x 3 f 2 (x 3 ).
b) Thit lp tp hp tt c cc gi tr a m f (ax) = af (x).
t S = {a > 0 : f (ax) = af (x), x R}.
R rng 1 S.
1

Ta chng t nu a S th a 3 S. Tht vy


axf 2 (x) = af (x3 ) = f (ax3 ) = f (a 3 x)3 = a 3 x.f 2 (a 3 x)


2

a 3 f 2 (x) = f 2 (a 3 x)
1

a 3 f (x) = f (a 3 x)
Nu a, b S th a + b S. Tht vy


f ((a + b)x) = f (a 3 x 3 )3 + (b 3 x 3 )3
1

= (a 3 + b 3 ) f 2 (a 3 x 3 ) f (a 3 x 3 ).f (b 3 x 3 ) + f 2 (b 3 x 3 )
2

= (a 3 + b 3 ) a 3 a 3 b 3 + b 3 x 3 f 2 (x 3 ) = (a + b)f (x).
Bng quy np ta chng t mi n N u thuc S. V bi ton ra l trng hp c bit vi n = 1996.
GV: Trn Minh Hin . . . . . . PTH bi dng hc sinh gii . . . . . . Trng THPT chuyn Quang Trung

www.VNMATH.com

3 PHNG PHP QUY NP

Nhn xt: 1. Nu ch n thun chng minh kt qu ca bi ton th c th quy np trc tip. Bng
cch kho st nh trn ta s thy ht c tt c cc gi tr ca a > 0 m f (ax) = af (x).
2. Do yu cu c bit ca bi ton, nn t nhin ta s ngh ngay l c th chng minh iu
ng vi mi s t nhin, v qua , s ngh ngay n hng quy np.
3. Vic suy ra du ca f (x) cng du vi x l quan trng, n gip ta trit tiu bnh phng m
khng cn xt du, y cng l mt iu ng lu trong rt nhiu bi tp khc.
4. Bi ton trn rt c th xut pht t hng ng thc x3 + y 3 = (x + y) (x2 xy + y 2 ).
V d 3.5. Tm tt c cc hm f : Z Z tha mn:
f (x3 + y 3 + z 3 ) = f 3 (x) + f 3 (y) + f 3 (z), x, y, z Z
Hint:
1. Tnh f (0) v chng minh f l hm l.
2. Chng t f (2) = 2f (1), f (3) = 3f (1). Chng minh bng quy np f (n) = nf (1), n Z 3. Trong
chng minh chuyn t n = k 0 sang n = k + 1, ta s dng hng ng thc sau: Nu k chn th k = 2t,
ta c:
(2t + 1)3 + 53 + 13 = (2t 1)3 + (t + 4)3 + (4 t)3 khi k = 2t
v nu k l th k = 2t 1 khi n = 2t lun c vit di dng 2t = 2j (2i + 1), v ng thc trn ch
cn nhn cho 23j
V d 3.6. Tm tt c cc hm f : N N tha mn cc iu kin:
f (1) > 0 v f (m2 + n2 ) = f 2 (m) + f 2 (n), m, n N
Hint:
1. Tnh f (0) f (m2 + n2 ) = f (m2 ) + f (n2 )
2. Chng minh f (n) = n, n 10. Vi n > 10 ta s dng cc ng thc sau:
(5k + 1)2 + 22 = (4k + 2)2 + (3k 1)2
(5k + 2)2 + 12 = (4k + 1)2 + (3k + 2)2
(5k + 3)2 + 12 = (4k + 3)2 + (3k + 1)2
(5k + 4)2 + 22 = (4k + 2)2 + (3k + 4)2
(5k + 5)2 = (4k + 4)2 + (3k + 3)2

GV: Trn Minh Hin . . . . . . PTH bi dng hc sinh gii . . . . . . Trng THPT chuyn Quang Trung

www.VNMATH.com
4 KHAI THC TNH CHT N
NH, TON NH, SONG NH, CHN L CA HM S

Khai thc tnh cht n nh, ton nh, song nh, chn l
ca hm s
Trc tin ta nhc li cc khi nim c bn ny.

a) Nu f : R R l n nh th t f (x) = f (y) ta suy ra c x = y.


b) Nu f : R R l ton nh th vi mi y R, tn ti x R f (x) = y.
c) Nu f : R R l song nh th ta c c hai c trng trn.
Nu mt hm s m n nh chng ta rt hay dng th thut tc ng f vo c hai v, nu mt hm
f ton nh ta hay dng: Tn ti mt s b sao cho f (b) = 0, sau tm b. Nu quan h hm l hm bc
nht ca bin v phi th c th ngh ti hai quan h ny.
V d 4.1. Tm tt c cc hm s f : Q Q tha mn
f (f (x) + y) = x + f (y), x, y Q.
Gii
Nhn xt, hm ng nht 0 khng tha mn bi ton. Xt f (x) 6 0.
a) f n nh, tht vy, nu f (x1 ) = f (x2 ) th
f (f (x2 ) + y) = f (f (x2 ) + y) x1 + f (y) = x2 + f (y) x1 = x2 .
b) f ton nh, tht vy, v tn ti y0 sao cho f (y0 ) 6= 0. Do v phi ca iu kin l mt hm s bc
nht ca x nn c tp gi tr l Q.
c) Tnh f (0), cho x = y = 0 v s dng tnh n nh ta c
f (f (0)) = f (0) f (0) = 0.
T thay y = 0 ta c
f (f (x)) = x, x Q.
d) Thay x bi f (x) v s dng kt qu trn(v iu ny ng cho vi mi x Q v f l ton nh) th
f (x + y) = f (x) + f (y), x, y Q.
T y ta c f (x) = ax thay vo bi ton ta nhn f (x) x hoc f (x) x trn Q.
Nhn xt: Nu yu cu bi ton trn tp R th cn thm tnh cht n iu hoc lin tc. C th,
cc bn c th gii li bi ton sau (THTT, 2010): Tm tt c cc hm s lin tc f : R R tha
mn iu kin
f (x + f (y)) = 2y + f (x), x, y R.
V d 4.2. Tm tt c cc hm s f : R R tha mn
f (xf (y) + x) = xy + f (x), x, y R.
GV: Trn Minh Hin . . . . . . PTH bi dng hc sinh gii . . . . . . Trng THPT chuyn Quang Trung

www.VNMATH.com
4 KHAI THC TNH CHT N
NH, TON NH, SONG NH, CHN L CA HM S
Gii
Thay x = 1 vo iu kin hm ta c
f (f (y) + 1) = y + f (1), y R.
T y suy ra f l mt song nh. Ly x = 1, y = 0 ta c
f (f (0) + 1) = f (1) f (0) = 0 do f n nh.
By gi vi x 6= 0, t y =

f (x)
thay vo iu kin hm ta c
x

f (xf (y) + x = 0 = f (0)) xf (y) = x do f n nh,


hay f (y) = 1, tc l

f (x)
f
x

= f (y) = 1 = f (b),

vi b l mt s thc no (do f l mt ton nh). Vy f (x) = bx, x 6= 0. Kt hp vi f (0) = 0


th vit gp thnh f (x) = bx, x R. Thay vo iu kin hm s ta c c hai hm tha mn l
f (x) x v f (x) x.
Nhn xt: Bi ton ny c th gii bng cch th bin nh sau m khng cn dng n tnh song
nh ca hm s. Thay x = 1 ta c
f (f (y) + 1) = y + f (1), y R.
V d 4.3. ( ngh IMO 1988) Xc nh hm s f : N N tha mn iu kin sau:
f (f (n) + f (m)) = m + n, m, n N.

(14)

Gii
a) Trc tin ta kim tra f n nh. Tht vy gi s f (n) = f (m), khi
f (2f (n)) = f (f (n) + f (n)) = 2n,
v
f (2f (n)) = f (f (m) + f (m)) = 2m.
Do m = n, nn f n nh.
b) Ta tnh f (f (n)) theo cc bc sau: cho m = n = 0 trong (14) th ta c f (2f (0)) = 0, li cho
m = 2f (0) vo trong (14) th ta c
f (f (n)) = n + 2f (0).

GV: Trn Minh Hin . . . . . . PTH bi dng hc sinh gii . . . . . . Trng THPT chuyn Quang Trung

www.VNMATH.com
4 KHAI THC TNH CHT N
NH, TON NH, SONG NH, CHN L CA HM S
c) Tc ng f vo c hai v ca (14) v s dng kt qu trn, ta c
f (f (f (n) + f (m))) = f (n) + f (m) + 2f (0).
Ngoi ra theo quan h bi th
f (f (f (n) + f (m))) = f (n + m).
T y ta c
f (n + m) = f (n) + f (m) + 2f (0).
Cho m = n = 0 th f (0) = 0, do quan h trn tr thnh hm cng tnh. Vy f (n) = an. Thay
vo quan h bi ton ta c
f (n) = n, n N.
- Nhn xt: Quan h n nh ca bi ton ny khng cn thit trong li gii. V bi ton ny c th
chng minh bng quy np trn N.
Cch 2. Nu xt trn Z+ th ta c th chng minh bng quy np f (x) = x, x N. Tc l, dng
phng php, ta chng minh khng cn tn ti hm s no khc. Trc tin ta tnh f (1). Gi s
f (1) = t > 1, t s = f (t 1) > 0. Nhn thy rng nu f (m) = n th
f (2n) = f (f (m) + f (m)) = 2m.
Nh vy
f (2t) = 2, f (2s) = 2t 2.
Nhng khi th
2s + 2t = f (f (2s) + f (2t)) = f (2t) = 2 t < 1,
iu ny v l. Vy f (1) = 1. Gi s ta c f (n) = n th
f (n + 1) = f (f (n) + f (1)) = n + 1.
Vy f (n) = n, n Z+ .
V d 4.4. (Balkan 2000) Tm tt c cc hm s f : R R tha mn iu kin:
f (xf (x) + f (y)) = (f (x))2 + y, x, y R.

(15)

Gii
a) Ta tnh f (f (y)) bng cch cho x = 0 vo (15) ta c
f (f (y)) = (f (0))2 + y, y R.
b) Chng t f n nh. Tht vy nu f (y1 ) = f (y2 ) th f (f (y1 )) = f (f (y2 )). T y theo phn (a) th
f 2 (0) + y1 = (f (0))2 + y2 y1 = y2 .
c) Chng t f ton nh v v phi ca (15) l mt hm bc nht ca y nn c tp gi tr bng R. Kt
hp hai iu trn ta thu c f l mt song nh t R vo R.
GV: Trn Minh Hin . . . . . . PTH bi dng hc sinh gii . . . . . . Trng THPT chuyn Quang Trung

www.VNMATH.com
4 KHAI THC TNH CHT N
NH, TON NH, SONG NH, CHN L CA HM S
d) Tnh f (0). Da vo tnh ton nh th phi tn ti a R f (a) = 0. Thay x = y = a vo (15) ta
c
f (af (a) + f (a)) = (f (a))2 + a f (0) = a.
Do f l mt song nh nn a = 0, tc f (0) = 0. T y theo (a) th
f (f (x)) = x, x R.
Trong (15) cho y = 0 ta c
f (xf (x)) = (f (x))2 , x R.

(16)

Trong quan h trn, thay x bi f (x) ta c(thay c ng vi mi x R v f l song nh)


f (f (x).f (f (x))) = [f (f (x))]2 , x R
f (f (x) x) = x2 , x R
(f (x))2 = x2 , x R.
T y suy ra vi mi x R th hoc l f (x) = x hoc l f (x) = x. Chng ta chng t l phi c s
ng nht f (x) = x, x R hoc l f (x) = x, x R ch khng th xy ra s an xen gia hai gi tr.
Tht vy, gi s tn ti a 6= 0, n 6= 0 sao cho f (a) = a, f (b) = b th khi trong quan h (15) thay
x = a, y = b ta c

f a2 + b = a2 + b.
Nhng v gi tr ca f (a2 + b) ch c th l a2 + b hoc l a2 b. Nhng nhn thy a2 + b khng th
bng vi mt gi tr no trong hai gi tr trn. Vy iu gi s l sai.
Kim tra li thy hai hm s f (x) = x, x R hoc l f (x) = x, x R tha mn yu cu bi ton.
V d 4.5. (IMO 1992) Tm tt c cc hm s f : R R tha mn iu kin

f x2 + f (y) = (f (x))2 + y, x, y R.

(17)

Gii
a) f n nh, tht vy nu f (y1 ) = f (y2 ) th

f x2 + f (y1 ) = f x2 + f (y2 ) (f (x))2 + y1 = (f (x))2 + y2 y1 = y2 .


b) f ton nh, v v tri lm hm bc nht theo y nn f c tp gi tr l ton b R. Kt hp hai iu
trn suy ra f l mt song nh.
c) Tnh f (0). Do f song nh nn tn ti duy nht a R sao cho f (a) = 0. Thay x = 0 ta c
f (f (y)) = (f (0))2 + y.
Thay x = y = a vo (17) v s dng kt qu trn, ta c

f a2 = a

f (a) = f f a2

0 = (f (0))2 + a2
f (0) = a = 0.
GV: Trn Minh Hin . . . . . . PTH bi dng hc sinh gii . . . . . . Trng THPT chuyn Quang Trung

www.VNMATH.com
4 KHAI THC TNH CHT N
NH, TON NH, SONG NH, CHN L CA HM S
T y ta thu c quan h quen thuc

f (f (x)) = x, x R v f x2 = (f (x))2 (thay y = 0).


T y th nu x 0 th f (x) 0, ngoi ra f (x) = 0 khi v ch khi x = 0. By gi ly x 0, y R th
f (x + y) = f

+ f (f (y)) = f

+ f (y) = f

2 

+ f (y),

hay
f (x + y) = f (x) + f (y), x 0, y R.
Thay y = x ta c f (x) = f (x) hay f l hm l. Do nu x < 0 th
f (x + y) = f ((x y)) = f (x y) = f (x) f (y) = f (x) + f (y), y R, x < 0.
Kt hp hai iu trn ta thu c quan h cng tnh ca hm f
f (x + y) = f (x) + f (y), x, y R.
Ngoi ra s dng tnh cht f (x) = 0 khi v ch khi x = 0 ta cn c thm f n iu tng. Tht vy, vi
x > y th x y > 0 nn f (x y) > 0, do
f (x) = f ((x y) + y) = f (x y) + f (y) > f (y).
Hm f cng tnh v n iu nn c dng f (x) = ax, thay vo ta c a = 1. Vy f (x) = x, x R
tha mn bi ton.
V d 4.6. Tm tt c cc hm s f : R R tha mn
f (x + f (y)) = x + f (y) + xf (y), x, y R.
Gii
Ta c th vit li quan h hm di dng
f (x + f (y)) = (f (y) + 1) x + f (y), x, y R.

(18)

a) Nu f (x) 1, d dng kim tra hm ny tha mn.


b) Xt f (x) khng ng nht 1. Khi phi tn ti y0 R f (y0 ) 6= 1. Khi v phi ca (18)
l hm bc nht ca x nn c tp gi tr l R. iu ny chng t f l ton nh.
Cho x = 0 ta thu thm c mt quan h na l
f (f (x)) = f (x), x R.
Khi vi mi x R, do f ton nh nn s tn ti y(ph thuc vo x) sao cho x = f (y), khi
f (x) = f (f (y)) = f (y) = x.
Tuy nhin, thay hm ny vo (18) th khng tha mn.

GV: Trn Minh Hin . . . . . . PTH bi dng hc sinh gii . . . . . . Trng THPT chuyn Quang Trung

www.VNMATH.com
4 KHAI THC TNH CHT N
NH, TON NH, SONG NH, CHN L CA HM S
Kt lun: Hm s tha mn yu cu bi ton l f (x) 1.
V d 4.7. (Vit Nam TST 2004) Tm tt c cc gi tr ca a, sao cho tn ti duy nht mt hm
s f : R R tha mn iu kin

f x2 + y + f (y) = (f (x))2 + ay, x, y R.

(19)

Gii
Nhn thy nu a = 0 th c hai hm s tha mn l f (x) 0 v f (x) 1. Do ta xt trng hp
a 6= 0.
a) Hm f ton nh. Tht vy do v phi l hm bc nht ca y nn c tp gi tr l R. Do f ton
nh, khi tn ti b R sao cho f (b) = 0.
b) f (x) = 0 khi v ch khi x = 0. Thay y = b vo (19) ta c

f x2 + b = (f (x))2 + ab.
T phng trnh trn ta thy

(20)

f x2 + b = (f (x))2 + ab.
Do ta c (f (x))2 = (f (x))2 hay |f (x)| = |f (x)| , x R. T iu ny ta thu c thm
f (b) = 0. Li thay y = b vo (19) ta c

f x2 b = (f (x))2 ab.

(21)

T (20) v (21) ta nhn c

f x2 + b f x2 b = 2ab, x R.
Thay x = 0 vo ng thc trn ta c 2ab = f (b) f (b) = 0 b = 0. Vy f (x) = 0 x = 0.
c) a = 2. Trong (19) cho y = 0 th f (x2 ) = (f (x))2 , x R. T y cho x = 1 ta c f (1) =
(f (1))2 f (1) = 1(v f (1) 6= 0 do phn (b)). Li trong (19) cho y = 1 th c

f x2 + 2 = (f (x))2 + a = f x2 + a.
Thay x = 0 vo ng thc trn th a = f (2). Do vy
a2 = (f (2))2
= f (22 ) = f (4)


= f ( 2)2 + 2
= f (2) + a = 2a.
Vy a = 2 v a 6= 0.

GV: Trn Minh Hin . . . . . . PTH bi dng hc sinh gii . . . . . . Trng THPT chuyn Quang Trung

www.VNMATH.com
4 KHAI THC TNH CHT N
NH, TON NH, SONG NH, CHN L CA HM S
By gi ta gii phng trnh hm

f x2 + y + f (y) = (f (x))2 + 2y, x, y R.


Thay y =

(22)

(f (x))2
vo (22) ta c
2
(f (x))2
f x
2

(f (x))2
+f
2

= 0, x R.

V tnh cht ca f l f (x) = 0 khi v ch khi x = 0 nn


(f (x))2
f
2

= x2 +

(f (x))2
, x R.
2

Li trong (22) v s dng kt qu trn ta c

f x2 y 2 = (f (x))2 (f (y))2 = f (x2 ) f (y 2 ), x, y R.


T ng thc ny cho x = 0 th f (y 2 ) = f (y 2 ) tc f l hm l. Nn quan h trn c th vit li
di dng
f (x + y) = f (x) + f (y), x, y R.
Li s dng (f (x))2 = f (x2 ) th (f (x + y))2 = f ((x + y)2 ), khai trin v s dng tnh cng tnh ta c
f (xy) = f (x)f (y), x, y R.
Hm f va cng tnh, va nhn tnh nn f (x) x. Th li thy hm s ny tha mn bi.
Nhn xt: Mt phn ca bi ton trn xut hin u tin trn tp ch AMM, c xut bi
Wu Wei Chao, v c chn l mt bi ton chn i tuyn Bungari nm 2003 v chn i tuyn Iran
2007, trong ch gii quyt cho trng hp a = 2.
V d 4.8. ( ngh IMO 2002) Tm tt c cc hm s f : R R tha mn
f (f (x) + y) = 2x + f (f (y) x) , x, y R.
Gii
a) f ton nh, tht vy thay y = f (x) ta c
f (f (f (x)) x) = f (0) 2x, x R.
Do v phi l hm bc nht ca x nn c tp gi tr l R.
b) V f ton nh nn tn ti a sao cho f (a) = 0. Thay x = a vo bi th
f (y) a = f (f (y) a) + a.
V f ton nh nn quan h trn c th vit li
f (x) = x a vi a l hng s.
GV: Trn Minh Hin . . . . . . PTH bi dng hc sinh gii . . . . . . Trng THPT chuyn Quang Trung

www.VNMATH.com
4 KHAI THC TNH CHT N
NH, TON NH, SONG NH, CHN L CA HM S
Th li thy hm s ny tha mn.
V d 4.9. (THTT T8/360). Tm tt c cc hm s f : R+ R+ tha mn
f (x).f (y) = f (x + yf (x)) , x, y R+ .

(23)

Gii
Gi s f l hm s tha mn bi ton.
a) Nu f (x) (0, 1), x R+ th khi thay y =

f (x)f

x
1 f (x)

x
vo (23) ta c
1 f (x)

=f

x
, x R+ ,
1 f (x)

suy ra f (x) = 1, tri vi gi thit f (x) (0, 1). Vy gi tr ca hm s f lun ln hn hoc bng 1.
b) Nu tn ti gi tr a R+ sao cho f (a) = 1, th khi thay x = a ta c
f (y + a) = f (y), y R+ .
Ngoi ra, ng vi mi x R+ c nh v h R+ cho trc, lun tn ti y R+ yf (x) = h. Do
f (x + h) = f (x + yf (x)) = f (x)f (y) f (x).
Kt hp hai iu trn bt buc phi c f (x) 1. Kim tra li thy hm s ny tha mn.
c) Nu f (x) > 1, x R+ th f n nh. Tht vy, khi
f (x + h) = f (x + yf (x)) = f (x)f (y) > f (x), x, h R+ .
Chng t f l hm ng bin ngt trn R+ , do n l mt n nh trn R+ . i vai tr ca x v
y trong (23) ta c
f (y + xf (x)) = f (x + yf (x)) , x, y R+ .
V f n nh nn
y + xf (y) = x + yf (x), x, y R+ .
T y ta c
f (x) 1
f (y) 1
=
, x, y R+ ,
x
x
y
y
hay
f (x) 1
= a, x R+ f (x) = ax + 1, a > 0.
x
x
Th li thy hai hm s f (x) 1 hoc f (x) = ax + 1, a > 0, x R+ tha mn bi ton.
V d 4.10. (USA 2002) Tm tt c cc hm s f : R R tha mn

f x2 y 2 = xf (x) yf (y), x, y R.
Gii
GV: Trn Minh Hin . . . . . . PTH bi dng hc sinh gii . . . . . . Trng THPT chuyn Quang Trung

www.VNMATH.com
4 KHAI THC TNH CHT N
NH, TON NH, SONG NH, CHN L CA HM S
a) f (0) = 0 (thay x = y = 0).
b) f l hm l, tht vy

xf (x)yf (y) = f (x)2 y 2 = f (x2 y 2 ) = xf (x)yf (y), x, y R f (x) = f (x), x 6= 0.


T y ta ch tnh ton vi x, y 0.
c) f (x) = f (x y) + f (y) (1). Cho x = 0 ta c f (x2 ) = xf (x), thay vo quan h hm ta c

f x2 = f x2 y 2 + f y 2 f (u) = f (u v) + f (v), u, v 0.
d) f (2t) = 2f (t), ch cn thay x = 2t, y = t vo (1).
e) Tnh f (2t + 1) theo hai cch, trc tin vi x = t + 1, y = 1 th vo (1) ta c
f (t + 1) = f (t) + f (1).
Thay x = t + 1, y = t vo iu kin ban u cng vi s dng kt qu trn, ta c
f (2t + 1) = (t + 1)f (t + 1) tf (t) = f (t) + (t + 1)f (1).
Ngoi ra, thay 2t + 1, y = 1 vo trong (1) ta c
f (2t + 1) = f (2t) + f (1) = 2f (t) + f (1).
Kt hp hai kt qu trn ta c
2f (t) + f (1) = f (t) + (t + 1)f (1) f (t) = tf (1), t 0.
Vy f (x) = ax, x R, a l hng s. Kim tra li thy hm s ny tha mn.
Nhn xt: 1. Quan h (1) l quan h cng tnh. Tuy nhin nu ta dng tnh cng tnh y th ch
thu c kt qu trn Q. V gi thit ca bi ton khng th khai thc thm c tnh cht lin tc
hoc n iu nn khng th c c kt qu hm f (x) = ax. Cch tnh f (2t + 1) theo hai cch trn
l mt tng hay, mang t tng ca cch tnh sai phn.
2. Nu bi ton c thm gi thit (f (x))2 = f (x2 ) th bng cc khai trin (f (x + 1))2 = (f (x + 1))2
theo tnh cht cng tnh, ta thu c quan h nhn tnh, t bi ton d dng gii hn.
V d 4.11. Tm tt c cc hm s f : R R tha mn iu kin:
f ((1 + x)f (y)) = yf (f (x) + 1) , x, y R.
Gii
R rng nhn t quan h hm ta thy nu hm s f l n nh th bi ton tr nn rt d dng. Tht
vy nu hm f n nh th thay y = 1 ta c
f ((1 + x)f (1)) = f (f (x) + 1) , x R.
GV: Trn Minh Hin . . . . . . PTH bi dng hc sinh gii . . . . . . Trng THPT chuyn Quang Trung

www.VNMATH.com
4 KHAI THC TNH CHT N
NH, TON NH, SONG NH, CHN L CA HM S
T y do f n nh nn (1 + x)f (1) = f (x) + 1, hay f (x) c dng hm s bc nht f (x) = ax + b.
Thay li vo quan h hm ta c a = 1, b = 0. Vy trong trng hp ny c hm s f (x) = x, x R
tha mn bi ton.
Vn cn li l nu hm f khng n nh. Tc l tn ti y1 6= y2 m f (y1 ) = f (y2 ). Khi ta c
y1 f (f (x) + 1) = f ((1 + x)f (y1 )) f ((1 + x)f (y2 )) = y2 f (f (x) + 1) , x, R.
T iu trn th phi c f (f (x) + 1) = 0, x R. Thay vo quan h hm ta phi c f ((1 + x)f (y)) =
0, x, y R. Nu tn ti y0 sao cho f (y0 ) 6= 0 th ta c f ((1 + x)f (y0 )) = 0, x R hay f (x) 0(mu
thun). Vy chng t khng tn ti y0 f (y0 ) 6= 0, tc f (y) 0. T y ta c hm ng nht f (x) 0
tha mn bi ton.
Nhn xt: Quan h n nh trong bi ton ny chnh l im mu cht ca li gii.
V d 4.12. Xc nh tt c cc hm s f : N N, ng thi tha mn hai iu kin:
f (2) = 2 v f (mn) = f (m)f (n), m, n N
V d 4.13. Tm tt c cc hm s lin tc f : R R tha mn iu kin:
f (xf (y)) = yf (x), x, y R
Hint:
1. Nhn thy f (x) 0 tha mn. Xt f (x) 6= 0.
2. Kim tra f n nh, cng vi f lin tc, f (1) > f (0) nn f tng ngt.
3. Tc ng f vo hai v, so nh f (xf (y)) v xf (y).
p s: f (x) = x, x R.
Nhn xt: Bi ton ny cng c th dng php th thch hp a v hm nhn tnh
V d 4.14. Tm tt c cc hm s f : Z+ Z+ tha mn:
f (f (n) + m) = n + f (m + 2003), m, n Z+ .
Gii
a) Trc tin ta chng minh f n nh. Tht vy nu f (n1 ) = f (n2 ) th
f (f (n1 ) + m) = f (f (n2 ) + m)
n1 + f (m + 2003) = n2 + f (m + 2003) n1 = n2
b) Thay m = f (1) ta c
f (f (n) + f (1)) = n + f (f (1) + 2003)
= n + 1 + f (2003 + 2003)
= f (f (n + 1) + 2003)
V f n nh nn f (n) + f (1) = f (n + 1) + 2003 hay f (n + 1) = f (n) + f (1) 2003. iu ny dn n
f (n + 1) f (n) = f (1) 2003, tc f (n) c dng nh mt cp s cng, vi cng sai l f (1) 2003,
s hng u tin l f (1). Vy f (n) c dng f (n) = f (1) + (n 1) (f (1) 2003), tc f (n) = an + b.
Thay vo quan h hm ta c f (n) = n + 2003, n Z+ .
GV: Trn Minh Hin . . . . . . PTH bi dng hc sinh gii . . . . . . Trng THPT chuyn Quang Trung

www.VNMATH.com
5 KHAI THC TNH N IU CA HM S

Khai thc tnh n iu ca hm s

Trong mc ny, ta xt mt s bi ton gii phng trnh hm c s dng n tnh n iu ca hm


s. Mt s iu cn lu :
a) Nu f cng tnh v n iu trn R (hoc R+ ) th f (x) = kx.
b) Nu f n iu thc s th f l n nh.
c) Trong mt vi trng hp, nu ta d on c cng thc ca hm s chng hn f (x) = g(x) th c
th xt f (x) > g(x) v f (x) < g(x), sau s dng tnh n iu ca hm f dn ti iu v l.
d) Nu hm f n iu v ta c cng thc ca f trn tp s hu t Q th dng k thut chn hai
dy hu t n iu ngc nhau, ri sau chuyn qua gii hn.
V d 5.1. Tm tt c cc hm n iu f : R R tha mn
f (x + f (y)) = f (x) + y, x, y R.
Gii
a) Ta chng minh f n nh.
Tht vy gi s f (x1 ) = f (x2 ), khi vi mi x R ta c
f (x + f (x1 )) = f (x + f (x2 )) f (x) + x1 = f (x) + x2 x1 = x2 .
b) Li thay x = 0 th
f (f (y)) = f (0) + y hay f (f (x)) = x + f (0)x R.
c) By gi thay x = f (x) vo quan h hm th
f (f (x) + f (y)) = f (f (x)) + y = x + y + f (0) = f (0 + f (x + y)).
Do f n nh nn f (x+y) = f (x)+f (y), x, y R. V f n iu v cng tnh trn R nn f (x) = kx.
Thay vo quan h hm ta tm c k = 1. Vy f (x) = x hay f (x) = x l nhng hm s cn tm.
V d 5.2. Tm tt c cc hm tng nghim ngt f : R R tha mn
f (f (x) + y) = f (x + y) + 1, x, y R.
Gii
a) Tnh f (f (x)).
Cho y = 0 ta c
f (f (x)) = f (x) + 1.

GV: Trn Minh Hin . . . . . . PTH bi dng hc sinh gii . . . . . . Trng THPT chuyn Quang Trung

www.VNMATH.com
5 KHAI THC TNH N IU CA HM S
b) S dng tnh n iu ca hm s.
Thay x bi f (x) v s dng kt qu trn ta c
f (f (f (x)) + y) = f (f (x) + y) + 1 f (f (x) + 1 + y) = f (x + y) + 1 + 1.

(24)

Thay y bi f (y) ta c
f (f (x) + f (y)) = f (x + f (y)) + 1 = f (x + y) + 1 + 1.

(25)

T (24) v (25) ta c
f (f (x) + y + 1) = f (f (x) + f (y)).
V f l hm n iu nn
f (x) + y + 1 = f (x) + f (y) f (x) = x + 1, x R.
Th li thy hm s f (x) = x + 1, x R tha mn yu cu bi.
V d 5.3. (Hy lp 1997) Gi s f : (0, ) R tha mn ba iu kin:
(a) f tng nghim ngt.
(b) f (x) > x1 vi mi x > 0 v

(c) f (x)f f (x) +

1
x

= 1 vi mi x > 0.

Tnh f (1).
Gii
t t = f (1). Th x = 1 vo (c), ta c tf (t + 1) = 1. Do t 6= 0 v f (t + 1) = 1t . Li t x = t + 1
vo trong (c) ta c


1
f (t + 1)f f (t + 1) +
= 1.
t+1
Khi

1
1
f
+
= t = f (1).
t t+1

Do f l hm tng nghim ngt nn


1
1
+
= 1.
t t+1
Gii ra ta c t =

1 5
.
2

Nu t =

1+ 5
2

> 0, th

1 < t = f (1) < f (1 + t) =


mu thun. Do f (1) = t =

1 5
.
2

1
<1
t

Ch l hm s

1 5
f (x) =
2x

l mt hm tha mn yu cu bi ton.
GV: Trn Minh Hin . . . . . . PTH bi dng hc sinh gii . . . . . . Trng THPT chuyn Quang Trung

www.VNMATH.com
5 KHAI THC TNH N IU CA HM S
V d 5.4. Tm tt c cc hm s f : [1; +) [1; +) tha mn
f (xf (y)) = yf (x), x [1; +).
Gii
a) f l hm n nh. Tht vy nu f (y1 ) = f (y2 ) th
f (xf (y1 )) = f (xf (y2 ) y1 f (2x) = y2 f (2x), x [1; +) y1 = y2 .
(iu trn ng v min gi tr ca hm s nm trong [1; +), tc l khc 0).
b) Tnh f (1).
Cho x = y = 1 th f (f (1)) = f (1), do f n nh nn f (1) = 1.
c) Cho x = 1 th f (f (y)) = y.
d) Vi y > 1 th f (y) > 1(do f n nh). Vi x > y 1 th

f (x) = f

x
.y = f
y

x
.f (f (y)) = f (y).f
y

x
y

> f (y).

Suy ra f ng bin trn [1; +).


e) Ta chng minh f (x) = x, x [1; +). Tht vy, gi s c x0 [1; +) sao cho f (x0 ) 6= 0. Nu
f (x0 ) > x0 th
f (f (x0 )) > f (x0 ) x0 > f (x0 )
v l. Tng t cho trng hp ngc li. Vy f (x) = x, x [1; +) tha mn yu cu bi ton.
V d 5.5. (Iran 1997) Cho f : R R l hm gim tha mn
f (x + y) + f (f (x) + f (y)) = f [f (x + f (y)) + f (y + f (x))], x R.
Chng minh rng f (f (x)) = x, x R.
Gii
a) Lm xut hin f (f (x)).
Cho y = x ta c
f (2x) + f (2f (x)) = f (2f (x + f (x))) .

(26)

Thay x = f (x) trong vo trong (26) ta c


f (2f (x)) + f (2f (f (x))) = f (2f (f (x) + f (f (x)))) .

(27)

Ly (27) tr cho (26) ta c


f (2f (f (x))) f (2x) = f (2f (f (x) + f (f (x)))) f (2f (x + f (x))) .

(28)

GV: Trn Minh Hin . . . . . . PTH bi dng hc sinh gii . . . . . . Trng THPT chuyn Quang Trung

www.VNMATH.com
5 KHAI THC TNH N IU CA HM S
b) S dng tnh cht hm f l hm gim. Gi s tn ti x0 sao cho f (f (x0 )) > x0 , khi 2f (f (x0 )) >
2x0 . Do f l hm gim nn f (2f (f (x0 ))) < f (2x0 ). Do v tri ca (28) nh hn 0. Vy
f (2f (f (x0 ) + f (f (x0 )))) f (2f (x0 + f (x0 ))) < 0
f (2f (f (x0 ) + f (f (x0 )))) < f (2f (x0 + f (x0 ))) .
Li do f l hm gim nn
2f (f (x0 ) + f (f (x0 ))) > 2f (x0 + f (x0 )) f (x0 ) + f (f (x0 )) < x0 + f (x0 ) f (f (x0 )) < x0 .
iu ny dn n mu thun.
Nu tn ti x0 sao cho f (f (x0 )) < x0 th lp lun tng t nh trn ta cng dn n iu v l. Vy
f (f (x)) = x, x R.
V d 5.6. (Italy 2000)
a) Tm tt c cc hm n iu ngt f : R R tha mn
f (x + f (y)) = f (x) + y, x, y R.
b) Chng minh rng vi mi s nguyn n > 1, khng tn ti hm n iu ngt f : R R sao cho
f (x + f (y)) = f (x) + y n , x, y R.
Gii
Hm n iu ngt th n nh. Ngoi ra d thy f l mt song nh.
a) Th x = y = 0 ta c f (f (0)) = f (0). Do f n nh nn f (0) = 0. T ta c quan h
f (f (x)) = x, x R.
Khi vi mi z R, thay y = f (z) vo quan h hm ta c
f (x + z) = f (x) + f (z), x, z R.
T tnh cht cng tnh ca hm f v tnh n iu ngt ca f , suy ra f c dng f (x) = ax. Thay
vo hm ban u ta c f (x) x hoc f (x) x l hai hm tha mn yu cu bi ton.
Cch khc: Ta c th kim tra trc tip hai hm s ny nh sau. Xt trng hp f l hm tng, gi
s tn ti x0 R sao cho f (x0 ) > x0 , th do f l hm tng nn
f (f (x0 )) > f (x0 ) x0 > f (x0 ),
v l, tng t cho trng hp f (x0 ) < x0 . Vy f (x) x. Tng t cho hm gim.

GV: Trn Minh Hin . . . . . . PTH bi dng hc sinh gii . . . . . . Trng THPT chuyn Quang Trung

www.VNMATH.com
5 KHAI THC TNH N IU CA HM S
b) Trc tin ta khng nh n phi l s l. Tht vy v f n nh nn vi y 6= 0 th f (y) 6= f (y) dn
n
f (x + f (y)) 6= f (x + f (y)) f (x) + y n 6= f (x) + (y)n y n 6= (y)n .
Nu n chn th ng thc trn v l, vy n phi l s nguyn l.
Lp lun tng t nh phn (a) ta vn thu c
f (0) = 0, f (f (x)) = xn , x R.
Tc ng f vo c hai v ca ng thc cui ta c
f n (y) = f (f (f (y))) = f (y n ) .
Do ta thu c quan h di y
f (x + y n ) = f (x + f (f (y)))
= f (x) + f n (y)
= f (x) + f (y n ) , x, y R.

(29)

V do n l nn ta thu c tnh cng tnh f (x + y) = f (x) + f (y), x, y R. V f n iu nn c


dng f (x) = ax, thay vo ta thy khng tha mn. Vy khng tn ti hm s tha mn yu cu bi
ton.
Ta c th tip cn cch khc m khng qua tnh cng tnh ca hm s nh sau. Ta c f (f (1)) = 1.
Nu f l hm tng, th lp lun ging nh phn (a) ta c f (1) = 1. Khi
f (2) = f (1 + f (1)) = f (1) + 1n = 2,
v
2n = f (f (2)) = f (2) = 2,
mu thun. Nu f l hm gim th lp lun tng t.
V d 5.7. (APMO 1989) Xc nh tt c cc hm s f : R R tha mn ba iu kin di y:
(i) Hm f c tp gi tr l R.
(ii) Hm f tng ngt trn R.
(iii) f (x) + f 1 (x) = 2x, x R, trong f 1 l hm ngc ca f .
Gii
Ta d thy rng:
a) S tn ti hm ngc f 1 ca hm s f v f 1 : R R.
b) Hm s f 1 l hm tng.

GV: Trn Minh Hin . . . . . . PTH bi dng hc sinh gii . . . . . . Trng THPT chuyn Quang Trung

www.VNMATH.com
5 KHAI THC TNH N IU CA HM S
Vi iu kin (iii) ca bi ton ta c th kim tra d dng cc hm s c dng f (x) = x + c, trong
c l mt hng s ty , u tha mn tt c cc iu kin ca bi ton, v khi hm ngc ca f c
dng f 1 (x) = x c. Nh vy bi ton lun c nghim. Mc ch ca s trnh by di y l chng
t rng bi ton khng c nghim no khc ngoi nghim f (x) = x + c.
Bc 1. Vi mi s thc a R, ta xy dng tp S(a) = {x R|f (x) = x + a}. Mi s thc x R
u phi thuc mt tp S(a) no , bi v ly x0 R, th nhin nhin x0 S(a) vi a = f (x0 ) x0 .
Do vy ta thy rng tn ti t nht mt s thc a sao cho S(a) 6= .
Bc 2. Ta chng minh rng x0 S(a) x0 + ka S(a) vi mi k Z. Nh quy np, ta thy rng
ch cn chng minh x0 S(a) x0 + a S(a). Tht vy
Nu x0 S(a) f (x0 ) = x0 + a f 1 (x0 + a) = x0 . Do theo iu kin (iii) th
2 (x0 + a) = f (x0 + a) + f 1 (x0 + a) = f (x0 + a) + x0 ,
suy ra
f (x0 + a) = x0 + 2a = (x0 + a) + a x0 + a S(a).
Ngc li, nu x0 + a S(a) f (x0 + a) = x0 + 2a nn
2 (x0 + a) = f (x0 + a) + f 1 (x0 + a) = x0 + 2a + f 1 (x0 + a) f 1 (x0 + a) = x0 .
Do
f (x0 ) = x0 + a x0 S(a).
Bc 3. Ta chng minh rng nu S(a) 6= th S(b) = vi mi b 6= a.
Gi s b < a v x0 S(a). Vi y R ty , lun tn ti k Z sao cho
x0 + k(a b) y < x0 + (k + 1)(a b) (1),
suy ra
x0 + ka y + kb < (x0 + ka) + (a b).
Theo iu kin (ii) ca bi ton ta c
f (y + kb) f (x0 + ka) = x0 + (k + 1)a.
Do , nu y + kb S(b) f (y + kb) = y + (k + 1)b, suy ra
y + (k + 1)b x0 + (k + 1)a y x0 + (k + 1)(a b)
tri vi iu kin (1). Vy y + kb 6 S(b), do theo bc 2, ta c y 6 S(b). Chng t S(b) = .
Nu b > a th S(b) 6= , th theo phn trn ta li suy ra S(a) = , tri vi gi thit.
Kt lun: Nu S(a) 6= th vi mi x R u phi thuc mt tp S(b) no , m ta c S(b) = nu
b 6= a, do vy vi mi x R u phi cng thuc mt tp S(a), tc l ta c S(a) = x + a, x R.

GV: Trn Minh Hin . . . . . . PTH bi dng hc sinh gii . . . . . . Trng THPT chuyn Quang Trung

www.VNMATH.com
6 KHAI
THC TNH CHT IM BT NG CA HM S

Khai thc tnh cht im bt ng ca hm s

Cho hm s f : X R. im a X gi l im c nh(im bt ng, im kp) ca hm s f


nu f (a) = a.
Vic nghin cu cc im bt ng ca mt hm s cng cho ta mt s thng tin v hm s . im
bt ng a ca hm s f chnh l chu trnh bc 1 ca im a qua nh x f
V d 6.1. (IMO 1983) Tm cc hm s f : R+ R+ tha mn hai iu kin:
lim f (x) = 0 v f (xf (y)) = yf (x), x, y R+

Gii
a) Tnh f (1).
Cho x = y = 1, ta c f (f (1)) = f (1). Li cho y = f (1) ta c
f (xf (f (1))) = f (1)f (x) f (xf (1)) = f (1)f (x).
Mt khc f (xf (1)) = f (x) nn ta c
f (x) = f (x)f (1) f (1) = 1(do f (x) > 0).
b) im c nh ca hm s
Cho x = y vo quan h hm ta c
f (xf (x)) = xf (x), x R+ .
Suy ra xf (x) l im bt ng ca hm s f .
c) Mt s c im ca tp im c nh.
Nu x v y l hai im c nh ca hm s, th
f (xy) = f (xf (y)) = yf (x) = xy.
Chng t xy cng l im bt ng ca hm s. Nh vy tp cc im bt ng ng vi php nhn.
Hn na nu x l im bt ng th


1 = f (1) = f
Ngha l

1
1
1
1
f (x) = xf
f
= .
x
x
x
x

1
cng l im bt ng ca hm s. Nh vy tp cc im bt ng ng vi php nghch
x

o.
Nh vy ngoi 1 l im bt ng ra, nu c im bt ng no khc th hoc im bt ng ny
ln hn 1, hoc nghch o ca n ln hn 1. Do ly tha nhiu ln ca im ny ln hn 1 cng
s l im bt ng. iu ny tri vi iu kin th 2 trong quan h hm.
d) Vy 1 l im bt ng duy nht ca hm s, do xf (x) l im bt ng ca hm s vi mi x > 0
1
nn t tnh duy nht ta suy ra f (x) = .
x
D thy hm s ny tha mn yu cu bi ton.
GV: Trn Minh Hin . . . . . . PTH bi dng hc sinh gii . . . . . . Trng THPT chuyn Quang Trung

www.VNMATH.com
6 KHAI
THC TNH CHT IM BT NG CA HM S
V d 6.2. (IMO 1994) Gi s S l tp hp cc s thc ln hn 1. Tm tt c cc hm s f : S S
sao cho cc iu kin sau c tha mn
a) f [x + f (y)) + xf (y)] = y + f (x) + yf (x) x, y S

b)

f (x)
l hm thc s tng vi 1 < x < 0 v vi x > 0 .
x

Gii
a) Tm im bt ng.
T iu kin (b) ta nhn thy phng trnh im bt ng f (x) = x c nhiu nht l 3 nghim(nu
c): mt nghim nm trong khong (1; 0), mt nghim bng 0, mt nghim nm trong khong
(0; +).
b) Nghin cu im bt ng ca hm s.
Gi s u (1; 0) l mt im bt ng ca f . Trong iu kin (a) cho x = y = u ta c
f (2u + u2 ) = 2u + u2 .
Hn na 2u + u2 (1; 0) v 2u + u2 l mt im bt ng na ca hm s trong khong (1; 0).
Theo nhn xt trn th phi c
2u + u2 = u u = u2 (1; 0).
Hon ton tng t, khng c im bt ng no nm trong khong (0; +). Nh th 0 l im bt
ng duy nht ca hm s(nu c).
c) Kt lun hm
Cho x = y vo (a) ta c
f (x + f (x) + xf (x)) = x + f (x) + xf (x),

x S.

Nh vy vi mi x S th x + (1 + x)f (x) l im bt ng ca hm s. Theo nhn xt trn th


x
, x S.
x + (1 + x)f (x) = 0, x S f (x) =
1+x
Th li thy hm ny tha mn yu cu bi ton.
V d 6.3. (IMO 1996) Tm tt c cc hm s f : N N sao cho:
f (m + f (n)) = f (f (m)) + f (n),

m, n N.

Gii
a) Tnh f (0).
Cho m = n = 0 th ta c
f (f (0)) = f (f (0)) + f (0) f (0) = 0.
T y li cho n = 0 th f (f (m)) = f (m), m N. Vy ta c quan h hm nh sau
8
<

f (m + f (n)) = f (m) + f (n) (1)


.
f (0) = 0
(2)

GV: Trn Minh Hin . . . . . . PTH bi dng hc sinh gii . . . . . . Trng THPT chuyn Quang Trung

www.VNMATH.com
6 KHAI
THC TNH CHT IM BT NG CA HM S
b) Nhn thy hm f (0) 0 tha mn yu cu bi ton.
c) Tm im c nh ca hm s.
Nu f khng ng nht 0. Th t quan h f (f (m)) = f (m), m N suy ra vi mi m N th f (m)
l im c nh ca hm s vi m N.
d) Tnh cht ca cc im bt ng.
Nu a v b l hai im bt ng ca hm s f th
f (a + b) = f (a + f (b)) = f (f (a)) + f (b) = f (a) + f (b) = a + b.
Vy tp cc im bt ng bt bin qua php cng.
e) Tp hp cc im bt ng ca f . Gi a l im bt ng khc 0 b nht ca hm s f .
- Nu a = 1, tc l f (1) = 1, th d thy rng f (2) = 2 (bng cch cho m = n = 1). V p dng
phng php quy np ta suy ra f (n) = nn N.
- Nu a > 1, tc l f (a) = a. Bng phng php quy np ta cng chng t c l f (ka) = ka, k 1.
Ta chng minh tp cc im bt ng ng u c dng ka, k 1(lu l a l im bt ng nh
nht ca hm s). Tht vy nu n l im bt ng khc th n = ka + r(0 r < a), khi theo (1)
v tnh cht im bt ng ca ka, ta c
n = f (n) = f (ka + r) = f (r + f (ka)) = f (r) + f (ka) = f (r) + ka f (r) = n ka = r.
V r < a m r li l im bt ng, a l im bt ng nh nht, nn r = 0. Chng t cc im bt
ng u c dng ka, k 1 (*).
f) Xy dng hm f .
V {f (n) : n N} l tp cc im bt ng ca hm f . Vy th vi i < a th do (*) nn ta c
f (i) = ni a vi n0 = 0, ni N.
Ly s nguyn dng n bt k th ta c th vit n = ka + i(0 i < a). Theo quan h u bi th
f (n) = f (i + ka) = f (i + f (ka)) = f (i) + ka = ni a + ka = (ni + k)a.
Ta kim chng hm f nh vy tha mn yu cu bi ton. Tht vy, vi m = ka + i, n = la + j , 0
i, j < a th
f (m + f (n)) = f (la + j + f (ka + i)) = ka + i + f (f (la + i)) .

GV: Trn Minh Hin . . . . . . PTH bi dng hc sinh gii . . . . . . Trng THPT chuyn Quang Trung

www.VNMATH.com
6 KHAI
THC TNH CHT IM BT NG CA HM S
V d 6.4. (AMM, E984) Tm tt c cc hm s f : R R sao cho
f (f (x)) = x2 2, x R.
Gii
Ta chng minh mt kt qu tng qut hn: Cho S l mt tp hp v g : S S l mt hm s c chnh
xc 2 im c nh {a, b} v g g c chnh xc 4 im c nh {a, b, c, d}. Th khng tn ti hm s
f : S S g = f f .
Chng minh
Gi s g(c) = y. Th c = g (g(c)) = g(y), nn y = g(c) = g (g(y)). Do vy y l mt im c nh ca
g g. Nu y = a th a = g(a) = g(y) = c, dn n mu thun. Tng t cho y = b s dn n mu
thun l c = b. Nu y = c th c = g(y) = g(c), tc c l im c nh ca g, mu thun. T suy ra
y = d, tc l g(c) = d, v tng t th g(d) = c.
Gi s tn ti f : S S sao cho f f = g. Th f g = f f f = g f . Khi f (a) =
f (g(a)) = g (f (a)), nn f (a) l mt im c nh ca g. Bng vic kim tra tng trng hp ta kt lun
f {a, b} = {a, b}, f {a, b, c, d} = {a, b, c, d}.
Xt f (c). Nu f (c) = a, th f (a) = f (f (c)) = g(c) = d, mu thun do f (a) nm trong {a, b}. Tng
t cng khng th xy ra f (c) = b. Ngoi ra cng khng th c f (c) = c v c khng l im c nh ca
g. Do vy ch c kh nng l f (c) = d. Nhng khi th
f (d) = f (f (c)) = g(c) = d,
mu thun, v iu ny khng th xy ra do d khng phi l im c nh ca g. Do vy khng th tn
ti hm f tha yu cu bi ton.
Quay tr li bi ton, bi ton l trng hp c bit ca hm g(x) =
x2 2, c
hai im c nh
5
1
5
1
+
2
v
. p dng kt
l 1 v 2, v g (g(x)) = (x2 2) 2 c cc im c nh l 1, 2,
2
2
qu trn ta hon thnh li gii cho bi ton.

GV: Trn Minh Hin . . . . . . PTH bi dng hc sinh gii . . . . . . Trng THPT chuyn Quang Trung

www.VNMATH.com
7 PHNG PHP A V PHNG TRNH SAI PHN

Phng php a v phng trnh sai phn

Khi cn xc nh cc hm s f (n) t N vo R th ta c th t yn = f (n) v a phng trnh hm


cho v phng trnh sai phn v s dng kin thc ca l thuyt phng trnh sai phn.
V d 7.1. Tm tt c cc hm s f : N R tha mn iu kin:
f (n)f (m) = f (n + m) + f (n m), n, m N, n m
Hint:
1. Tnh f (0). Nu f (0) = 0 f (n) 0. Nu f (0) = 2
2. t m = 1, khi t a = f (1), xn = f (n) a v phng trnh:
x0 = 2, x1 = a, xn+2 axn+1 + an = 0, (n 1)
Ngay c khi cn tm hm s f : X X vi X R nhng trong phng trnh hm cho l hm
hp ta cng c th s dng l thuyt phng trnh sai phn.
Xt hm s f : X X. Xt dy cc hm s (fn )n vi n N, fn : X X c xc nh nh sau:
V d 7.2. Cho a, b R+ . Tm tt c cc hm s f : R+ R+ tha mn iu kin:
f (f (x)) + af (x) = b(a + b)x, x R+
Hint:
1. Vi mi x R+ , ta xy dng dy s xn nh sau:
x0 = x, x1 = f (x); fn+1 = f (fn ), n N(xn 0)
ta c phng trnh sai phn:
xn+2 + axn+1 b(a + b)xn = 0
2. Gii phng trnh c trng tm xn = bn + (1)n (a + b)n , v sao = 0
p s: f (x) = x
V d 7.3. Tm tt c cc hm s f : [0, 1] [0, 1] tha mn iu kin:
f (2x f (x)) = x, x [0, 1]
Hint:
Khai thc hm g(x) = 2x f (x) th gn (x) = n(g(x) x) + x
p s: f (x) = x
V d 7.4. Xc nh cc hm s f : N R tha mn iu kin:
f (0) = 1, f (1) = 2, f (n + 1)f 2 (n 1) = f 3 (n), n N
Hint:
1. Nhn xt rng f (n) > 0, n N.
2. Ly ln hai v
n
p s: f (n) = 22 1
GV: Trn Minh Hin . . . . . . PTH bi dng hc sinh gii . . . . . . Trng THPT chuyn Quang Trung

www.VNMATH.com
7 PHNG PHP A V PHNG TRNH SAI PHN
V d 7.5. Xc nh hm s f : N R tha mn phng trnh hm:

f (0) = 2, f (n + 1) = 3f (n) +
Hint:
Chuyn v ri bnhphng.
n
8)
+
p s: f (n) = (8+ 66)(3+
8

8f 2 (n) + 1, n N

(8 66)(3 8)n
8

Nhn xt: Cc bi ton trn c ngun gc t dy s, tng l tuyn tnh ha dy s chuyn qua
thnh tuyn tnh ha dy hm. V thc cht th dy s l mt loi hm c bit
V d 7.6. Tm cc hm s f : N N tha mn:
f (n + 3) f (n + 1) = f (n) + f (n + 2), n 1
Hint:
1. t an = f (n) suy ra: an+4 an = an+3 (an+5 an+1 )
2. Tnh a4 a0 theo a3 , a3 , . . . , an
3. rng bn s hng lin tip ca an khng th tt c u bng 1(mu thun vi gi thit). Suy ra
dy tun hon
vi chu k 4.

a0 + a2 = a1 a3
4. Gii h
p s: (a0 , a1 , a2 , a3 ) = (2, 2, 2, 2) = (3, 1, 2, 5) = (2, 1, 3, 5) = (2, 5, 3, 1) =
a1 + a3 = a0 a2
(3, 5, 2, 1)
V d 7.7. Tm tt c cc hm s f : N N tha mn:
f (n) + f (n + 1) = f (n + 2)f (n + 3) 2000, n N

GV: Trn Minh Hin . . . . . . PTH bi dng hc sinh gii . . . . . . Trng THPT chuyn Quang Trung

www.VNMATH.com
8 PHNG
PHP S DNG TNH LIN TC CA HM S

Phng php s dng tnh lin tc ca hm s

i vi hm s lin tc chng ta thng s dng tnh cht: Nu (xn ) x, f l hm lin tc th


f (xn ) f (x)
Nu f : [a, b] R th f b chn trn on [a, b]
Nu f lin tc v n nh th f l hm n iu.
Hm s f : R R lin tc th ta k hiu f C[R]_tp cc hm lin tc trn R.
Trong loi ton ny cng hay p dng t tng: nu ta cn chng minh hm l hm hng th chng
t n l hm hng trn mt dy s, ri s dng tnh lin tc suy ra n l hng s trn ton b tp
hp.
V d 8.1. Tm tt c cc hm s f C[R] tha mn

x2 f (y) + yf x2 = f (xy) + a, x, y R.
Gii
Cho y = 0 ta c
x2 f (0) = f (0) + a, x R.
iu ny ch c th xy ra khi f (0) = a = 0. Vy nu a 6= 0 th bi ton v nghim. Xt a = 0, khi
th

x2 f (y) + yf x2 = f (xy), x, y R.
Thay x = y = 1 ta c f (1) = 1. Li thay y = 1 v s dng f (1) ta c f (x2 ) = f (x), x R. n
y cho x = y th c
(x2 + x 1)f (x) = 0, x R.

Vy phi c f (x) = 0, x 6= 12
hm s ny tha mn bi.

. Nhng v f lin tc trn ton b R nn phi c f (x) 0. D thy

V d 8.2. Tm cc hm f C[R] tha mn iu kin:


f (4x) + f (9x) = 2f (6x), x R
Gii
a) a v dng s dng tnh lin tc.
t
t t = 6x x = . Ta c
6


2
3
2
3
f
t +f
t = 2f (t), t R f
t f (t) = f (t) f
t
3
2
3
2
hay

2
3
g
t = g(t) vi g(t) = f (t) f
t .
3
2

GV: Trn Minh Hin . . . . . . PTH bi dng hc sinh gii . . . . . . Trng THPT chuyn Quang Trung

www.VNMATH.com
8 PHNG
PHP S DNG TNH LIN TC CA HM S
b) S dng tnh lin tc.
Bng quy np d dng chng minh c


g(t) = g

2
3

n

t , t R, n N+ .

Do f (x) lin tc suy ra g(x) cng lin tc, cho n ta c




.g(t) = lim g
n

2
3

n

t = g(0) = f (0) f (0) = 0.

3
2
T f (t) = f
t , t R hay f
x = f (x), x R. Tng t ta c
2
3


f (x) = f

2
3

n

x f (0)(n ).

Vy f (x) = C, vi C l hng s tha mn bi.


V d 8.3. ( ngh IMO??) Tm tt c cc hm f C[R] tha mn iu kin:
f (x2 ) + f (x) = x2 + x, x R
Gii
a) a v dng s dng tnh lin tc.
t g(x) = f (x) x v ta chng minh g l hm hng. Tht vy, g(x) lin tc trn R v
g(x2 ) + g(x) = 0, x R.
Thay x = 0 ta c g(0) = 0.
Thay x = 1 ta c g(1) = 0.
Thay x bi x th
g[(x)2 ] + g(x) = 0 g(x) = g(x2 ) = g(x).
Do g l hm chn nn ta ch cn xt trn min x > 0. T quan h hm trn ta suy ra g(x) =
g(x2 ) = g(x4 ) hay vi x > 0 th


1
g(x) = g x 4 .
b) S dng tnh lin tc.
Ly a > 0 ty , xt dy s (xn ) c xc nh nh sau
1

x0 = a, xn+1 = xn4 , n = 0, 1, 2, . . . .
Khi th lim xn = 1 v ta c
n

g(xn+1 ) = g xn4 = g(xn ) = g(xn1 ) = = g(x0 ) = g(a).


Vy g l hm hng trn dy (xn ). Theo tnh lin tc ca hm g th


g(a) = n
lim g(xn ) = g n
lim xn = g(1) = 0.
Vy g(x) 0, x R. Nh vy f (x) = x, x R tha mn bi.
GV: Trn Minh Hin . . . . . . PTH bi dng hc sinh gii . . . . . . Trng THPT chuyn Quang Trung

www.VNMATH.com
8 PHNG
PHP S DNG TNH LIN TC CA HM S
V d 8.4. ( ngh IMO 1992) Tm tt c cc hm s f : R+ R+ tha mn, vi a, b R+
f (f (x)) + af (x) = b(a + b)x, x R+ .
Gii
y l mt v d c in ca loi bi ton gii c bng s dng quan h hi quy. Vi mi x0 R+ ,
t
u0 = x0 , u1 = f (u0 ) , un+1 = f (un ) , n = 1, 2, 3, . . .
T quan h bi ton ta c quan h hi quy ca dy nh sau
un+2 = a.un+1 + b(a + b)un .
Xt phng trnh c trng
X 2 + aX b(a + b) = 0 X1 = bv X2 = (a + b).
Do

"

un = c1 b + c2 (1) (a + b) = (a + b)

c1

b
a+b

+ (1) c2 .

n
b
V lim
= 0 nn nu c2 > 0 th un < 0 vi n l ln, cn nu c2 < 0 th un < 0 vi n
a+b
chn ln. Trong c hai trng hp u dn n mu thun. Vy c2 = 0. Do un = c1 bn , cng vi
u0 = x0 = c1 . Do ta c u1 = f (x0 ) = bx0 . Do x0 bt k nn kt lun f (x) = bx, x R+ tha mn
bi ton.

V d 8.5. (Bulgari 1997) Tm cc hm s lin tc f : R R tha mn




f (x) = f x2 +

1
, x R.
4

Gii
Ta chng minh ch c hm hng tha mn yu cu bi ton. Trc tin ta d dng nhn thy f l hm
s chn nn ta ch cn xt trn min x 0. Ly a 0 bt k, xt hai trng hp
8

<x0 = a
1
a) Nu 0 a , th xt dy s (xn ) : :
1
2
xn+1 = x2n +
4


th

f (xn ) = f x2n1 +

1
= f (xn1 ) = = f (x0 ) = f (a).
4

1
1
1 1
1
Dy (xn ) b chn trn, v x1 = x20 + = a2 + + = v dng phng php quy np ta cng
4
4
4 4
2
1
chng minh c xn .
2
Dy (xn ) l dy tng, v


xn+1 xn =

x2n

1
1
+ xn = xn
4
2

2

0 xn+1 xn .

GV: Trn Minh Hin . . . . . . PTH bi dng hc sinh gii . . . . . . Trng THPT chuyn Quang Trung

www.VNMATH.com
8 PHNG
PHP S DNG TNH LIN TC CA HM S
T dy (xn ) c gii hn hu hn, t lim xn = b, khi


lim xn+1 = lim x2n +

1
1
1
b = b2 + b = .
4
4
2

V hm s f lin tc nn


1
f (a) = lim f (a) = lim f (xn ) = f (lim xn ) = f
= const.
2
8
<
x0 = a
1

b) Nu a > th xt dy s (xn ) : :
2
xn+1 = xn


1
+
= f (xn ) = = f (x0 ) = f (a).
4

Dy (xn ) b chn di, v x1 =


1
minh c xn .
2

1
v
4

x2n+1

f (xn+1 ) = f

, khi xn = x2n+1 +

1
4

1
x0 >
4

1 1
1
= , v bng phng php quy np ta chng
2 4
2

Dy (xn ) n iu gim, v


xn+1 xn = xn+1

x2n+1

1
1
= xn+1
4
2

2

0 xn+1 xn .

T dy (xn ) c gii hn hu hn, t lim xn = b, ta c




lim xn = lim x2n+1 +

1
1
1
b = b2 + b = .
4
4
2

V hm f lin tc nn


1
f (a) = lim f (a) = lim f (xn ) = f (lim xn ) = f
= const.
2


1
= const, a 0 hay f l hm hng trn x 0. V f l
Trong c hai trng hp, ta thy f (a) = f
2
hm chn nn f (x) = const l hm s duy nht tha mn iu kin ny.
Nhn xt: im mu cht trong li gii trn l: chng minh f (a) khng i, vi mi a 0, phi
xy dng dy s c lin quan n a, c gii hn hu hn, v hm s phi khng thay i trn dy ny.
Bi ton tng qut hn bi ton trn c cho di y.
V d 8.6. (Putnam 1996) Cho c l s thc dng. Tm tt c cc hm s f : R R tha mn iu
kin

f (x) = f x2 + c , x R.
V d 8.7. (VMO 2001) Cho hm s g(x) =

2x
. Hy tm tt c cc hm s f (x) xc nh v lin
1 + x2

tc trn khong (1, 1) v tha mn h thc

1 x2 f (g(x)) = 1 + x2

.f (x), x, y (1, 1).

GV: Trn Minh Hin . . . . . . PTH bi dng hc sinh gii . . . . . . Trng THPT chuyn Quang Trung

www.VNMATH.com
8 PHNG
PHP S DNG TNH LIN TC CA HM S
Gii
Vit li h thc cho di dng
2

(1 x2 )
2
f
(g(x))
=
1

x
.f (x), x, y (1, 1).
(1 + x2 )2

t (x) = (1 x2 ) f (x), x, y (1, 1). Khi f (x) lin tc trn (1, 1) v tha mn bi khi v
ch khi (x) lin tc trn (1, 1) v tha mn h thc
(g(x)) = (x), x, y (1, 1).

(30)

1x
, x (0, +) l mt song nh t(0, +) n (1, 1). Do , c th vit li h
1+x
thc trn nh sau
 



1x
1x
=
, x (0, +),
g
1+x
1+x
hay



1 x2
1x

=
, x, y (0, +).
1 + x2
1+x


1x
. Khi (x) lin tc trn (1, 1) v tha mn (30) khi v ch khi h(x)
Xt hm s h(x) =
1+x
lin tc trn (0, +) v tha mn h thc h (x2 ) = h(x), x (0, +). Bng phng php quy np,
d dng chng minh c

h(x) = h 2n x , x (0, +), n N.

a
, x (1, 1),
Do lim 2n x = 1 v h l hm lin tc nn h(x) h(1). T x = const v f (x) =
1 x2
vi a l hng s. Kim tra li thy hm s ny tha mn.

D thy u(x) =

V d 8.8. Tm tt c cc hm s f : R R tha mn ng thi hai tnh cht sau


a) f lin tc trn R.

b) f (x + m) f (x) +

m + 1 = (m + 2), x R, vi m l s nguyn dng.

Gii
Gi s tn ti hm s f lin tc trn R v tha mn iu kin

f (x + m) f (x) + m + 1 = (m + 2), x R.

Khi f (x) 6= 0 v f (x) 6= m + 1 trn R. V f lin tc trn R nn ch c th xy ra mt trong 3


trng hp i vi min gi tr ca f (k hiu l Imf ) nh sau:

a) Nu Imf (, m + 1) th

f (x + m) f (x) + m + 1 > 0 > (m + 2), x R.

GV: Trn Minh Hin . . . . . . PTH bi dng hc sinh gii . . . . . . Trng THPT chuyn Quang Trung

www.VNMATH.com
8 PHNG
PHP S DNG TNH LIN TC CA HM S

b) Nu Imf ( m + 1, 0) th m + 1 < f (x + m) < 0 nn

|f (x + m)| < m + 1
v
0 < f (x) +
Do

f (x + m) f (x) +

m+1<

m + 1.

m + 1 < m + 1 < m + 2, x R.

c) Nu Imf (0, +) th

f (x + m) f (x) +

m + 1 > 0 > (m + 2), x R.

Xt c ba trng hp ta thy khng tn ti hm s tha mn yu cu bi ton.


Nhn xt: Gn y, trong tp ch THTT thng 9 nm 2009 gii quyt bi ton ny trong trng
hp c bit m = 2008.
V d 8.9. Cho t (0, 1). Tm tt c cc hm f C[R] tha mn iu kin:
f (x) 2f (tx) + f (t2 x) = x2 , x R
Hint:
t g(x) = f (x) f (tx). Thay bin x tx nhiu ln tm hm g(x).
x2
p s: f (x) = (1t
2 )2 + C vi C l hng s.
V d 8.10. Tm tt c cc hm s f C[R] tha mn iu kin:
2f (2x) = f (x) + x, x R
Hint:
1. Tm nghim ring f (x) = ax a = 31
2. t f (x) = g(x) + 31 x, dng tnh lin tc tm hm g.
p s: f (x) = 13 x
V d 8.11. Tm tt c cc hm s f C[R] tha mn iu kin:
3f (2x + 1) = f (x) + 5x, x R
Hint:
Tm hm ring di dng f (x) = ax + b
p s: f (x) = x 23
Bi ton ny cng c th gii trc tip bng cch th x

x1
2

V d 8.12. Tm hm s f : R R tha mn cc iu kin sau:


8
>
<
>
:

f (1) = 1
f (x) +f (y) = f (x + y), x, y R
f (x)f x1 = 1, x 6= 0

GV: Trn Minh Hin . . . . . . PTH bi dng hc sinh gii . . . . . . Trng THPT chuyn Quang Trung

www.VNMATH.com
8 PHNG
PHP S DNG TNH LIN TC CA HM S
Hint:
iu kin th 3 chota chng minh hm lin tc
1. Nhn xt f (x) v f x1 cng du.
2. Chng minh nu |y| 2 th |f (y)| 2 v |y| 12 th |f (y)| 12
3. Chng minh |y| 21n th |f (y)| 21n , t hm lin tc ti 0.
p s: f (x) = x
V d 8.13. Tm cc hm s f C[R ] tha mn iu kin:
f (x3 ) x2 f (x) =

1
x, x, y R
x3

Hint:
3)
1. Vit li quan h hm thnh f (x
x16 = f (x)
x12
x3
x
x12 , dng tnh lin tc tm hm g.
2. t g(x) = f (x)
x
p s: f (x) = ax + x1 , a R

GV: Trn Minh Hin . . . . . . PTH bi dng hc sinh gii . . . . . . Trng THPT chuyn Quang Trung

www.VNMATH.com
9 NG DNG PHNG TRNH HM C BN

ng dng phng trnh hm c bn

V d 9.1. Tm tt c cc hm s lin tc f : [0, 1] R tha mn iu kin


f (x + y) = f (x) + f (y), x, y [0, 1].
Gii
Lp lun tng t ging nh phng trnh hm Cauchy ta c f (0) = 0 v


1
f (nx) = nf (x), n N, x 0,
.
n
T y ta c

1
1
f
= f (1).
n
n
By gi vi s hu t r [0, 1] lun c th vit di dng r =
ta c

f (r) = f

p
q

1
= f p.
q

= pf

1
q

p
vi p, q l cc s nguyn dng p q,
q

1
= p f (1) = rf (1).
q

Do vy f (x) = kx vi mi x [0, 1] Q. By gi vi s thc [0, 1], tn ti mt dy s hu t


{xn } [0, 1] sao cho lim xn = . Theo tnh lin tc ta c:
f () = lim f (xn ) = lim kxn = k lim xn = k.
Vy f (x) = kx, x [0, 1] vi k l s thc bt k.
V d 9.2. (VMO 2006) Hy tm tt c cc hm s lin tc f : R R tha mn iu kin
f (x y)f (y z)f (z x) + 8 = 0, x, y, z R.
Gii
t
t
Thay x = , y = , z = 0 vo quan h hm ta c
2
2
 

t 2
f (t) f
+ 8 = 0, t R.
2
T chng t f (x) < 0, x R. Vy ta c th t
f (x) = 2g(x) , x R, vi g(x) lm mt hm s lin tc trn R.
Thay vo ta c mi quan h ca hm g
g(x y) + g(y z) + g(z x) + 3 = 0, x, y, z R.
n y ta t h(x) = g(x) 1 th
h(u) + h(v) = h(u v), u, v R.
D thy h(0) = 0 v h(u) = h(u) nn ta nhn c
h(u + v) = h(u) + h(v), u, v R.
Hm h lin tc tha mn tnh cng tnh nn h(x) = ax, x R, t g(x) = ax + 1 v hm s
f (x) = 2ax+1 , x R, a l hng s. Kim tra li thy hm s ny tha mn bi ton.
GV: Trn Minh Hin . . . . . . PTH bi dng hc sinh gii . . . . . . Trng THPT chuyn Quang Trung

www.VNMATH.com
9 NG DNG PHNG TRNH HM C BN
V d 9.3. Tm tt c cc hm s f : R (0, +) lin tc v tha mn iu kin:
f (x2 + y 2 ) = f (x2 y 2 ) + f (2xy), x, y R
Hint:
Nhn xt hm l hm chn.

1. t a = x2 y2 , b = 2xy th vi a, b > 0 c tn ti x, y?? a v: f (a) + f (b) = f ( a2 + b2 )


2. t g(x) = f ( x)
p s: f (x) = kx2
V d 9.4. Tm tt c cc hm s f : R+ R lin tc tha:
f (xy) = xf (y) + yf (x), x, y R+
Hint:
t g(x) = f (x)
x
p s: f (x) = Cx ln x
V d 9.5. Tm tt c cc hm s f C[R] tha mn iu kin:
f (x + y) + f (z) = f (x) + f (y + z), x, y, z R
Hint:
1. Chuyn f (x + y) f (x) = f (y + z) f (z), v phi khng cha x nn v tri khng ph thuc vo x.
Vy f (x + y) f (x) = g(y)
2. Ta c g(x + y) = g(x) + g(y)
p s: f (x) = Cx + a
V d 9.6. Tm tt c cc hm lin tc f : R+ R+ tha mn:
f (f (xy) xy) + xf (y) + yf (x) = f (xy) + f (x)f (y), x, y R
Hint:
1. Cho y = 1 f [f (x) x] = f (1)[f (x) x]
2. Thay kt qu trn vo bi ton, li t g(x) = f (x) x c:
g(1)g(xy) = g(x)g(y)
p s: f (x) = x + Cxa vi C > 0 v a ty .
V d 9.7. Tm tt c cc hm s f : R R tha mn iu kin:
[f (x) + f (z)][f (y) + f (t)] = f (xy zt) + f (xt + yz), x, y, z, t R
Hint:
1. Xt trng hp f l hm hng.
2. Trng hp f
khng l hm hng, cho x = z = 0, lp lun thu c: f (x)f (y) = f (xy), x, y R
3. t g(x) = f ( x) th c: g(xy) = g(x)g(y) v g(x + y) = g(x) + g(y)
p s: f (x) = x2 , x R

GV: Trn Minh Hin . . . . . . PTH bi dng hc sinh gii . . . . . . Trng THPT chuyn Quang Trung

www.VNMATH.com
9 NG DNG PHNG TRNH HM C BN
V d 9.8. (APMO 2003) Tm tt c cc hm s f : R R tha mn hai tnh cht sau:
(i) Phng trnh f (x) = 0 ch c hu hn nghim.
(ii) f (x4 + y) = x3 f (x) + f (f (y)) , x, y R.
Gii
a) Tnh f (f (y)). Th x = 0 ta c
f (f (y)) = f (y), y R.
T y ta c quan h hm

f x4 + y = x3 f (x) + f (y).

(31)

b) Tnh f (0). Th x = 1, y = 0 vo (31) ta c f (0) = 0.


c) Li thay y = 0 vo (31) ta c f (x4 ) = x3 f (x). T y (31) tr thnh
f (x4 + y) = f (x4 ) + f (y), x, y R
hay
f (x + y) = f (x) + f (y), x 0, y R.
d) Th y = x vo quan h trn ta c f (x) = f (x) hay f l hm l. Do vi x < 0, y R th
f (x + y) = f (x y) = f (x) f (y) = f (x) + f (y).
Do vy ta chng t
f (x + y) = f (x) + f (y), x, y R.
T c im hm cng tnh, ta d thy nu f (x) l nghim th phi l f (x) = x. Tht vy, t tnh cht
cng tnh v quan h f (x) = f (f (x)) ta suy ra
f (f (x) x) = 0, x R.
Do phng trnh f (x) = 0 c hu hn nghim, m t trn th f (x) x lun l nghim ca phng
trnh. Do vy tp {f (x) x|x R} phi l hu hn.
By gi ta chng minh f (x) phi bng vi x. Gi s tn ti mt x0 R m f (x0 ) x0 6= 0. Th khi
vi k nguyn dng ta lun c
f (kx0 ) kx0 = kf (x0 ) kx0 = k (f (x0 ) x0 ) 6= 0, k Z+ .
M k (f (x0 ) x0 ) , k Z+ cha v hn gi tr, nn f (kx0 ) kx0 cng cha v hn gi tr, mu thun.
Vy hm s f (x) = x, x R tha mn yu cu bi ton.
Nhn xt: T quan h f (x4 ) = x3 f (x), dng phng php sai phn chng ta c th tm c
cng thc tng minh ca f (x). Trc tin ta k hiu 0g (x) = g(x), 1g (x) = 0g (x + 1) 0g (x),

GV: Trn Minh Hin . . . . . . PTH bi dng hc sinh gii . . . . . . Trng THPT chuyn Quang Trung

www.VNMATH.com
9 NG DNG PHNG TRNH HM C BN
2g (x) = 1g (x+1)1g (x), .... By gi ta s khai trin 3z (x) v 3s (x), vi r(x) = f (x4 ) v s(x) = x3 f (x)
bng cch s dng tnh cng tnh ca hm f .

1r = f (x + 1)4 f x4

= f x4 + 4x3 + 6x2 + 4x + 1 f x4

= 4f x3 6f x2 + 4f (x) + c,

2r = 4 f (x + 1)3 f x4

+ 6 f (x + 1)2 f x2

+ 4 (f (x + 1) f (x))

= 4 3f (x2 ) + 3f (x) + c + 6 (2f (x) + c) + 4c


= 12f (x2 ) + 24f (x) + 10c,

3r = 12 f (x + 1)2 f (x2 ) + 24 (f (x + 1) f (x))


1s

= 24f (x) + 36c


= (x + 1)3 f (x + 1) x3 f (x)
= (3x2 + 3x + 1)f (x) + c(x + 1)3 ,

2s = f (x + 1) 3(x + 1)2 + 3(x + 1) + 1 + c(x + 2)3


= (6x + 6)f (x) + c(6x2 + 18x + 14),

3s = (6(x + 1) + 6) f (x + 1) + c 6(x + 1)2 + 18(x + 1) + 14


= 6f (x) + 18xc + 36c.

V ta c 3r = 3r , x R bi v r(x) = s(x). T y ta c 24f (x) + 36c = 6f (x) + 18xc + 36c, suy ra


18f (x) = 18xc hay f (x) = cx. Thay vo quan h f (f (y)) = f (y) suy ra c = 0 hoc c = 1. Nu c = 1
th f (x) = x, nu c = 0 th f (x) 0 khng tha iu kin (i). Vy f (x) = x, x R l hm cn tm.
V d 9.9. (Mathematics Magazine) Tm tt c cc hm s f : R R tha mn
f (x + yf (x)) = f (x) + xf (y), x, y R.

(32)

Gii
Nhn xt: f (x) 0 tha mn bi ton. Xt trng hp hm f khng ng nht bng 0.
a) Tnh f (0). Thay y = 0, x = 1 ta c f (0) = 0. Ngoi ra nu f (x) = 0 th xf (y) = 0, y R, suy ra
x = 0. Vy
f (x) = 0 x = 0.
b) Quan h f (y + 1) = f (y) + f (1), y R. Thay x = 1 vo (32) ta c
f (1 + yf (1)) = f (1) + f (y), y R.
Nu f (1) 6= 1, th thay y =

1
vo phng trnh trn th
1 f (1)

1
1 f (1)

= f (1) + f

1
1 f (1)

f (1) = 0,

mu thun vi phn (a). Vy f (1) = 1. Do ta c quan h hm


f (1 + y) = 1 + f (y), y R.
T quan h ny ta c f (n) = n, n Z.
GV: Trn Minh Hin . . . . . . PTH bi dng hc sinh gii . . . . . . Trng THPT chuyn Quang Trung

www.VNMATH.com
9 NG DNG PHNG TRNH HM C BN
c) Tnh f (nx), n Z, x R. Thay x = n, y = z 1 ta c
f (nz) = f (n + (z 1)f (n)) = n + nf (z 1) = nf (z).
d) f cng tnh. Nu a = b th f (a) = f (b) = f (b) suy ra f (a) + f (b) = 0 =
f (a +b). Nu a 6= b

a+b
ab
a+b
th a + b 6= 0 v f (a + b) 6= 0(theo phn (a)). Thay x =
,y =
f
ta c
2
2
2
f (a) = f

ab
a+b

f
+
2
2f a+b
2

f (b) = f

ba
a+b
f

+
2
2f a+b
2

a+b
2
a+b
2

=f
!

=f

a+b
a+b
+
f
2
2

a+b
a+b
+
f
2
2

ab

,
2f a+b
2
!

ba

.
2f a+b
2

Cng hai ng thc trn, ta c

f (a) + f (b) = 2f

a+b
2

= f (a + b)(theo bc (c)).

e) f c tnh cht nhn tnh. p dng tnh cng tnh vo phng trnh hm ban u ta c
f (x) + f (yf (x)) = f (x + yf (x)) = f (x) + xf (y) f (yf (x)) = xf (y), x, y R.
Thay y = 1 ta c f (f (x)) = x, chng t f l mt song nh. Do li th z = f (x) vo quan h
trn ta c
f (yz) = xf (y) = f (z)f (y), y, z R.
f) Thay z = y ta c f (y 2 ) = f 2 (y) 0 v z = y ta c f (y 2 ) = f 2 (y) 0. Do f (a) > 0
khi v ch khi a > 0.
g) Thay y = 1 vo phng trnh hm (32) ta c
f (x f (x)) = f (x) x.
Do f (x) x v x f (x) i nhau, do theo bc (f) th x f (x) = 0, x R hay f (x) = x, x R.
Th li thy hm s ny tha mn yu cu.
V d 9.10. (THTT T7/231) Tm tt c cc hm s f : R R tha mn
f ((x + 1)f (y)) = y (f (x) + 1) , x, y R.
Gii
a) Tnh f (0) v f (1). Thay x = 1, y = 0 vo iu kin hm ta c
f (0) = y (f (1) + 1) , y R f (0) = 0, f (1) = 1.
T y cho x = 0 ta nhn c quan h
f (f (x)) = x, x R.
GV: Trn Minh Hin . . . . . . PTH bi dng hc sinh gii . . . . . . Trng THPT chuyn Quang Trung

www.VNMATH.com
9 NG DNG PHNG TRNH HM C BN
b) Tnh f (1). Thay y = f (1) v s dng kt qu trn ta c
f (x + 1) = f (1) (f (x) + 1) , x R.
T y th x = 2 v s dng f (1) ta c
1 = f (1) (f (2) + 1) .
Mt khc, thay x = 2, y = 1 vo iu kin ban u th
f (1) = (f (2) + 1) .
Kt hp hai ng thc ny th (f (1))2 = 1, do f (1) = 1, v nu f (1) = 1 th f (1) = f (f (1))
1 = 1(v l). T kt qu ca f (1), bng cch cho y = 1 ta nhn c quan h
f (x + 1) = f (x) + 1, x R.
c) f l hm nhn tnh, tht vy
f (xy) = f (x.f (f (y)))
= f ([(x 1) + 1] f (f (y)))
= f (y) (f (x 1) + 1) = f (x).f (y)
T y ta nhn c
f (x) = f

2

x. x = f
x 0, x 0,

v
f (x) = 0 x = f (f (x)) = f (0) = 0.
d) f l hm cng tnh, tht vy, vi y 6= 0 th

x
+ 1 .y
f (x + y) = f
y

x
=f
+ 1 f (f (y))
y

x
= f (y) f
+1
y

x
= f (y).f
+ f (y) = f (x) + f (y) do f nhn tnh.
y
Hm f va cng tnh, va nhn tnh nn f (x) = ax, thay vo ta c a = 1. Vy hm s tha mn bi
ton l f (x) = x, x R.
V d 9.11. (Belarus 1997) Tm tt c cc hm s f : R R tha mn iu kin
f (x + y) + f (x)f (y) = f (xy) + f (x) + f (y), x, y R.

(33)

Gii
Nu f (x) = c th t iu kin bi ton ta c f (x) 0 hoc f (x) 2. Xt f (x) khng phi l hm hng.
GV: Trn Minh Hin . . . . . . PTH bi dng hc sinh gii . . . . . . Trng THPT chuyn Quang Trung

www.VNMATH.com
9 NG DNG PHNG TRNH HM C BN
a) Tnh f (0). Thay y = 0 vo (33) ta c
f (x).f (0) = 2f (0), x R f (0) = 0,
do f (x) khc hm hng 2.
b) Tnh f (x + 2) theo hai cch. Trc tin thay y = 1 vo (33) ta c
f (x + 1) = (2 f (1)) f (x) + f (1) f (2) = (3 f (1)) f (1).
Thay x bi x + 1 v y = 1 ta c
f (x + 2) = (2 f (1)) f (x + 1) + f (1)
= (2 f (1))2 f (x) + (3 f (1))f (2)
= (2 f (1))2 f (x) + f (2).
Ngoi ra thay y = 2 vo (33) ta c
f (x + 2) = f (2x) + (1 f (2)) f (x) + f (2).
T hai ng thc trn ta c
f (2x) = (3 f (1)) f (x),
hay f (2x) = af (x) (vi a = 3 f (1) 6= 0(v nu khng hm f l hm hng 0), ngoi ra a 6= 1(nu
khng hm ng nht 2)) v f (4x) = a2 f (x).
c) Thay x bi 2x v y bi 2y v s dng kt qu trn ta c
af (x + y) + a2 f (x)f (y) = a2 f (xy) + af (x) + af (y), x, y R.
Ngoi ra, nhn c hai v ca (33) vi a2 ta c
a2 f (x + y) + a2 f (x)f (y) = a2 f (xy) + a2 f (x) + a2 f (y).
T hai ng thc ny ta thu c
a(a 1)f (x + y) = a(a 1) (f (x) + f (y)) , x, y R.
V a 6= 0, a 6= 1 nn ta c f (x + y) = f (x) + f (y), thay vo ta li c f (xy) = f (x)f (y). T hai
quan h ny ta c f (x) = x.
Kt lun: C ba hm s tha mn bi ton l f (x) 0, f (x) 2, f (x) = x, x R.
Nhn xt: 1. Phng trnh hm trn l tng ca hai phng trnh cng tnh f (x+y) = f (x)+f (y)
v hm nhn tnh f (xy) = f (x)f (y). Vi php th hp l nh trn ta a phng trnh hm v
li hm tha mn hai tnh cht trn.
2. Bi ton trn c s dng li trong Indian 2003 v trong rt nhiu k thi chn i tuyn ca
cc tnh nc ta. Nu trong (33) thay f (x) = g(x) 1 th ta c bi ton di y:
f (x + y) + f (xy) = f (x)f (y) + 1, x, y R.
V i chiu vi trn ta c hai hm s tha mn l f (x) 1 v f (x) = 1 + x, x R.
GV: Trn Minh Hin . . . . . . PTH bi dng hc sinh gii . . . . . . Trng THPT chuyn Quang Trung

www.VNMATH.com

10

10 BT NG THC HM

Bt ng thc hm

V d 10.1. (VMO 1994). Hy xc nh cc hm s f : R R tha mn


1
1
1
f (xy) + f (xz) f (x).f (yz) , x, y, z R.
2
2
4
Gii
a) Tnh f (0). Thay x = y = z = 0 ta c


1
1
1
1
f (0) + f (0) f 2 (0) f (0)
2
2
4
2

2

1
0 f (0) = .
2

1
b) Tnh f (1). Tng t nh trn bng cch thay x = y = z = 1 ta c f (1) = .
2
1
1
c) Chng t f (x) . Thay y = z = 0 v s dng f (0) = ta c
2
2
1
f (x) , x R.
2
1
1
d) Chng t f (x) . Thay y = z = 1 v s dng f (1) = ta c
2
2
1
f (x) , x R.
2
1
Vy ta c f (x) = , x R. Kim tra li thy hm ny tha mn yu cu.
2
V d 10.2. (Russian 2000) Tm tt c cc hm s f : R R tha mn iu kin
f (x + y) + f (y + z) + f (z + x) 3f (x + 2y + 3z), x, y, z R.
Gii
Thay y = z = 0 ta c
2f (x) + f (0) 3f (x), x R f (x) f (0), x R.
Li thay x = y =

x
x
, z = ta c
2
2
f (x) + 2f (0) 3f (0) f (x) f (0), x R.

T hai kt qu trn suy ra f (x) = f (x), x R hay f (x) c(c l hng s). Kim tra li thy hm s
ny tha mn.
V d 10.3. (THTT T8/230) Tm tt c cc hm s f : R R tha mn iu kin
|f (x) f (q)| 5(x q)2 , x R, q Q.
GV: Trn Minh Hin . . . . . . PTH bi dng hc sinh gii . . . . . . Trng THPT chuyn Quang Trung

www.VNMATH.com

10 BT NG THC HM

Gii
Vi mi x, x0 R(x x0 ), chn s hu t q nm gia x v x0 th
|f (x) f (x0 )| = |f (x) f (q) + f (q) f (x0 )|
|f (x) f (q)| + |f (q) f (x0 )| (bt ng thc tr tuyt i)
5(x q)2 + 5 (q x0 )2 (gi thit ca bi ton)
5 (x x0 )2 + 5 (x x0 )2 = 10 (x x0 )2 .
Vy nn lim |f (x) f (x0 )| = 0 hay lim f (x) = f (x0 ) . Suy ra f (x) l hm s lin tc ti mi x0 R.
xx0

xx0

Mt khc, t nh gi trn ta nhn c








f (x) f (x0 )
f (x) f (x0 )
10 |x x0 | lim
= 0,

xx0

x x0
x x0

hay f 0 (x0 ) = 0 vi mi x0 R. Do f (x) lin tc v c f 0 (x) = 0, x R, ta suy ra f (x) c, x R.


Th li thy hm s ny tha mn.
V d 10.4. (Nhn Bn 2007). Tm tt c cc hm s f : R+ R tha mn hai iu kin di y:
(i) f (x) + f (y)
(ii)

f (x + y)
, x, y R+ ,
2

f (x + y)
f (x) f (y)
+

, x, y R+ .
x
y
x+y

Gii
Ta ln lt khng nh cc d kin di y lin quan n hm s ny. Trc tin ta t hm s
f (x)
g(x) =
th t iu kin (ii) ta c
x
g(x) + g(y) g(x + y), x, y R+ .
a) g(nx) ng(x), n N, x R+ . iu ny d dng chng minh bng quy np da vo tnh cht ca
hm g. T y ta c
f (nx) n2 f (x), n N, x R+ .
b) f (2n x) = 4n f (x). Tht vy, trong (i) cho y = x ta c
4f (x) f (2x).
Tuy nhin theo phn (a) th f (2x) 4f (x). Do f (2x) = 4f (x), x R+ . T y ta thu c mt
c im ca g(x) l
g (2n x) = 2n g(x), n N, x R+ .

GV: Trn Minh Hin . . . . . . PTH bi dng hc sinh gii . . . . . . Trng THPT chuyn Quang Trung

www.VNMATH.com

10 BT NG THC HM

c) g(nx) = ng(x), n N, x R+ . Tht vy gi s tn ti mt n0 N, x0 R+ sao cho g(n0 x0 ) <


n0 g(x0 ). Khi chn r N sao cho 2r > n0 th
2r g(x0 ) = g (2r x0 )
= g (2r x0 + n0 x0 n0 x0 )
g (n0 x0 ) + g ((2r n0 ) x0 ) (tnh cht ca hm g)
< n0 g (x0 ) + g ((2r n0 ) x0 )
< n0 g (x0 ) + (2r n0 ) g(x0 ) (tnh cht ca hm g trong phn (a))
= 2r g (x0 ) (mu thun).
d) f (x) 0, x R+ . Tht vy,
10f (x) = 2 (f (x) + f (2x)) f (3x) 9f (x)texttheophn(a).
Do f (x) 0, ko theo
g(x) 0, x R+ .
e) g l hm n iu gim, v g(x + y) g(x) + g(y) g(x), x, y R+ .
f) g(x) = ax, v kt hp phn (c) th c ngay g(q) = g(1).q, q Q+ v g l hm n iu gim nn c
g(x) = ax, x R+ .
T ta suy ra f (x) = ax2 , x R+ . Th li hm s ny thy tha mn.
V d 10.5. (Eotvos - Kurschak 1979). Cho hm s f : R R tha mn
f (x) x v f (x + y) f (x) + f (y), x, y R.
Chng minh rng f (x) = x, x R.
Gii
Ta c f (0 + 0) f (0) + f (0) f (0) 0. Ngoi ra f (0) 0 nn ta c f (0) = 0. Vi mi x R th
0 = f (x + (x)) f (x) + f (x) x + (x) = 0,
do
f (x) + f (x) = 0 f (x) = f (x), x R.
Mt khc f (x) x nn x f (x) = f (x) x. Khi f (x) = x, x R.
V d 10.6. (Crux 2003). Tm tt c cc hm s f : R R tha mn

f x3 + x x (f (x))3 + f (x), x R.
Gii

GV: Trn Minh Hin . . . . . . PTH bi dng hc sinh gii . . . . . . Trng THPT chuyn Quang Trung

www.VNMATH.com

10 BT NG THC HM

Xt hm s g(x) = x3 + x, th hm g lin tc v tng ngt trn R, ngoi ra g(R) = R, do g l mt


song nh trn R. V vy tn ti hm ngc g 1 cng lin tc v tng ngt trn R. T iu kin bi
ton, c th vit li di dng
f (g(x)) x g (f (x)) , x R.
Thay x bi g 1 (x) vo bt ng thc f (g(x)) x ta c
f (x) g 1 (x), x R.
Tc ng hm g 1 vo bt ng thc x g (f (x)), vi ch l g 1 ng bin
g 1 (x) f (x), x R.
Kt hp hai bt ng thc trn ta c
f (x) = g 1 (x), x R.
D dng kim tra hm ny tha mn bi ton.
V d 10.7. Tm hm s f : R R tha mn iu kin:
f (x + y) f (x)f (y) 2002x+y , x, y R.
Gii
a) Tnh f (0), thay x = y = 0 ta c
f (0) (f (0))2 1 f (0) = 1.
b) Th y = x v s dng f (0) ta c
1 f (x).f (x) 1 f (x)f (x) = 1, x R.
c) Thay y = 0 ta c
f (x) 2002x , x R.
Nhng khi th
f (x).f (x) 2002x .2002x 1,
i chiu vi f (x).f (x) = 1, x R ta phi c f (x) = 2002x , x R.
Th li thy f (x) = 2002x , x R tha mn bi ton.
V d 10.8. (Bulgarian 1997) Tm hm s f : (0, +) (0, +) tha mn bt ng thc hm
(f (x))2 f (x + y)f (f (x) + y) , x, y > 0.
Gii

GV: Trn Minh Hin . . . . . . PTH bi dng hc sinh gii . . . . . . Trng THPT chuyn Quang Trung

www.VNMATH.com

10 BT NG THC HM

Ta chng minh f l hm gim.


Tht vy, vi x > y > 0 th tn ti s t > 0 sao cho x = y + t. Khi
f (y + t)
Do

(f (y))2
.
f (y) + t

(f (y))2
tf (y)
f (y) f (x) = f (y) f (y + t) f (y)
=
>0
f (y) + t
f (y) + t

(34)

hay f (x) < f (y). Vy f l hm gim. C nh x > 0, v ta chn s n N sao nf (x + 1) 1. Theo (34)
v kt hp vi f l hm gim ta c

k+1
k
f x+
f x+
n
n

1
f
n

k
n

x+

f x+

k
n

1
n

f x+

nf x +

k
n

k
n

+1

>

1
(k = 0, 1, 2, . . . , n 1).
2n

Cng cc bt ng thc trn ta c


f (x) f (x + 1) >

1
1
hay f (x + 1) < f (x) .
2
2

T y bng quy np ta c
f (x + 2m) < f (x) m, m N.
Ly m > f (x)(x c nh) th f (x + 2m) < 0, mu thun vi gi thit f (x) > 0, x > 0. Vy khng tn
ti hm s f tha mn yu cu bi ton.
Nhn xt: tng ca bi ton loi ny l c gng chng minh f (y) < 0 vi mt gi tr y > 0,
dn n mu thun. R rng chng ta ch cn chng minh l f (x) f (x + 1) c > 0 vi mi x bi v
n s dn n f (x) f (x + m) mc. Khi vi m ln th f (x + m) < 0. Li gii trn trnh by c
th t tng ny.
V d 10.9. (VMO 2003) Gi F l tp hp tt c cc hm f : R+ R+ tha mn bt ng thc
f (3x) f (f (2x)) + x, x R+ .
Tm s thc ln nht sao cho vi mi f F th f (x) .x, x R+ .
Gii
x
1
x
F , do . Hn na vi mi hm f F ta c f (x) . tng
2
2
3
1
gii quyt nh sau: K hiu = 1 v to mt dy {n } f (x) n x v mong mun dy ny tin
3
1
1
1
ti . iu ny s suy ra , v do = . By gi chng ta s xy dng quan h hi quy cho
2
2
2
k . Gi s rng f (x) k x, x R+ . Th t iu kin bt ng thc
R dng hm s f (x) =

f (3x) f (f (2x)) + a k f (2x) + x k .k .2x + x = k+1 .3x.


2n2 + 1
1
. By gi chng ta phi chng minh lim n = . Nhng y l bi
3
2
1
ton d dng, v d dng chng minh c dy k l dy tng v b chn trn bi . Do phi hi t
2
22 + 1
1
v gii hn tha mn =
tc = (v < 1).
3
2
iu ny c ngha l n+1 =

GV: Trn Minh Hin . . . . . . PTH bi dng hc sinh gii . . . . . . Trng THPT chuyn Quang Trung

www.VNMATH.com

11

11 HM TUN HON

Hm tun hon

V d 11.1. Tm tt c cc gi tr a R sao cho tn ti duy nht hm s f : R R tha mn


f (x f (y)) = a (f (x) x) f (y), x, y R.
Gii
t g(x) = f (x) x. Gi thit ca bi ton vit theo hm g l
g (x y g(y)) = ag(x) x, x, y R.
Gi s f (y) = g(y) + y khng phi l hm hng. Ly r, s l hai phn t phn bit trong min gi tr
ca f (y) = y + g(y). Khi vi mi x, y R th
g(x r) = ag(x) x = g(r s).
iu ny suy ra g(x) l hm tun hon vi chu k T = |r s| > 0. Khi
ag(x) x = g (x y g(y))
= g (x + T y g(y))
= ag (x + T ) (x + T )
= ag(x) x T
T iu ny suy ra T = 0, mu thun. Do vy f l hm hng. Tc l f (y) = c, c R. Thay vo quan
h hm ta c
a = 0, c = 0.

GV: Trn Minh Hin . . . . . . PTH bi dng hc sinh gii . . . . . . Trng THPT chuyn Quang Trung

www.VNMATH.com
12 MT S CHUYN PHNG TRNH HM

12

Mt s chuyn phng trnh hm

12.1

Phng trnh hm gii nh tnh gi tr hm s theo hai cch khc


nhau

V d 12.1. (THTT 11/394) Tm tt c cc hm s f : R R tha mn


f (f (x) + y) = f (x + y) + xf (y) xy x + 1.
Gii
Thay y = 0 vo quan h hm ta c
f (f (x)) = f (x) + xf (0) x + 1, x R.
V thay x = 0 vo ng thc trn ta c
f (f (0)) = f (0) + 1.
Thay y bi f (y) vo quan h hm ban u ta c
f (f (x) + f (y)) = f (x + f (y)) + xf (f (y)) xf (y) x + 1
= [f (x + y) + yf (x) xy y + 1] + x [f (y) + yf (0) y + 1] xf (y) x + 1
= f (x + y) + yf (x) + xyf (0) 2xy y + 2.
Hon chuyn vi tr x v y trong kt qu trn ta c
f (f (x) + f (y)) = f (x + y) + xf (y) + xyf (0) 2xy x + 2.
T y ta nhn c
yf (x) y = xf (y) x, x, y R.
Thay x = 0, y = 1 ta c f (0) = 1, do f (f (0)) = 2. Li thay y = 1 v s dng kt qu
f (f (0)) = 2, f (0) = 1 ta c
f (x) = x + 1, x R.
Th li thy hm s ny tha mn yu cu.
V d 12.2. Tm tt c cc hm s f : Q+ Q+ tha mn hai iu kin:
1. f (x + 1) = f (x) + 1, x Q+
2. f (x3 ) = f 3 (x), x Q+
Gii
Quy np ta chng minh c f (x + n) = f (x) + n, x Q+ , n N (*).
Vi mi s thc r = pq Q+ .
- Tnh theo cch (*) c: f (r + q 2 )3 = f 3 (r) + 3p2 + 3pq 3 + q 6
- Tnh theo iu kin (b) c: f (r + q 2 )3 = f 3 (r) + 3f 2 (r)q 2 + 3f (r)q 4 + q 6
T hai iu kin trn ta c:
q 2 f 2 (r) + q 4 f (r) (p2 + pq 3 ) = 0
GV: Trn Minh Hin . . . . . . PTH bi dng hc sinh gii . . . . . . Trng THPT chuyn Quang Trung

www.VNMATH.com
12.1 Phng trnh hm gii nh tnh gi
tr hm
12 sMT
theo hai
S cch
CHUYN
khc nhau
PHNG TRNH HM
Gii phng trnh ny ta tm c: f (r) = r, r Q+
Nhng t kt qu ca hm s f (x) = x, x Q+ th ta thay iu kin (b) cho hai n:
f (x3 + y) = x3 + y = f 3 (x) +

f (xy)
f (x)

v iu kin cng c m rng t Q+ thnh R+ vy ta c bi ton:


Bi tp tng t: Tm tt c cc hm f : R+ R+ tha mn:
f (x3 + y) = f 3 (x) +

f (xy)
, x, y R+
f (x)

Vn gian nan nht ca m rng ny l tnh c f (1).


Cho y = 1 ta c:
f (x + 1) = f 3 (x) + 1(1)
Cho x = 1 ta c:
f (y + 1) = f 3 (1) +

f (y)
(2)
f (1)

t f (1) = a th s dng (1) ta tnh c:


f (2) = a3 + 1, f (9) = (a3 + 1)3 + 1
S dng (2) ta tnh c:
f (9) = a3 + a2 + a + 1 +

1
1
1
1
1
+ 2+ 3+ 4+ 7
1 a
a
a
a

Gii phng trnh:


(a3 + 1)3 + 1a3 + a2 + a + 1 +

1
1
1
1
1
+ 2+ 3+ 4+ 7
1 a
a
a
a

ta c: a=1. Vy ta c:
f (x + 1) = f (x) + 1 v f (x3 ) = f 3 (x)
Tc l ta c c bi ton ban u. Cn vn s l trn R+ ta s dng tnh tr mt ca tp s thc
vi ch f l hm tng.

GV: Trn Minh Hin . . . . . . PTH bi dng hc sinh gii . . . . . . Trng THPT chuyn Quang Trung

13www.VNMATH.com
GII PHNG TRNH HM BNG CCH THM BIN

13

Gii phng trnh hm bng cch thm bin

V d 13.1. (Indian TST 2004) Tm tt c cc hm s f : R R tha mn


f (x + y) = f (x)f (y) c sin x sin y, x, y R,
trong c l hng s ln hn 1.
Gii
Bng cch thm bin mi z ta c
f (x + y + z) = f (x)f (y + z) c sin x sin(y + z)
= f (x) [f (y)f (z) c sin y sin z] c sin x[siny cos z + sin z cos y]
= f (x)f (y)f (z) cf (x) sin y sin z c sin x sin y cos z c sin x cos y sin z.
Nhng r rng f (x + y + z) = f (y + x + z), do ta c
sin z [f (x) sin y f (y) sin x] = sin z [cos x sin y cos y sin x] .
Th z =

ta nhn c
f (x) sin y f (y) sin x = cos x sin y cos y sin x.

Vi x = v y khng phi l bi nguyn ca , ta nhn c sin y [f () + 1] = 0, v do f () = 1.


Li thay x = y = 2 vo iu kin ban u ta c

f () = f
dn n f

2
c,
2

= c 1. Li th y = vo iu kin ban u ta c f (x + ) = f (x). Khi th


f (x) = f (x + ) = f x +
f
c cos x
2
2

=f
f (x)f
c sin x c cos x,
2
2
suy ra

f (x)
T iu ny suy ra f (x) = f
f (x) =

(35)

1 = cf
sin x c cos x.
f
2
2

sin x + cos x. D dng th li hai hm sau tha mn iu kin bi ton

c 1 sin x + cos x v f (x) = c 1 sin x + cos x.

GV: Trn Minh Hin . . . . . . PTH bi dng hc sinh gii . . . . . . Trng THPT chuyn Quang Trung

www.VNMATH.com
14 LUYN TP PHNG TRNH HM

14

LUYN TP PHNG TRNH HM

14.1

Phng php th bin

V d 14.1. Gii cc phng trnh hm sau


a) f : R R tha f (x + 1) = x2 + 2x + 3,


x+1
= x + 3,
b) f : R R tha f
x1

x 6= 1.

c) f : R R tha f (cos x) = sin2 x + 2,




x R.

1
1
= x3 + 3 ,
d) f : R R tha f x +
x
x


x R.

x 6= 0.

x1
1
e) f : R R tha f
+ 2f
= x,
x
x


x 6= 0, x 6= 1.

3+x
x3
+f
= x,
f) f : R R tha f
x+1
1x

14.2

x 6= 1.

Bt ng thc hm

V d 14.2. Gii cc bt phng trnh hm sau


a) f : R R tha [f (x) f (y)]2 |x y|3 ,

x, y R.

b) f : R R tha f (x3 + x) x [f (x)]3 + f (x),

x R.

c) f : [0, 1] R tha mn hai iu kin sau


(i) f (0) = f (1) = 0.

x + y
f (x) + f (y),
(ii) f
2

x, y [0, 1].

V d 14.3. K hiu X = R+ {0}, cho hm s f : X X v b chn trn on [0, 1] v tha mn


bt ng thc


y
x
f (x)f (y) x2 f
+ y2f
, x, y X.
2
2
Chng minh rng f (x) x2 ,

x X.

GV: Trn Minh Hin . . . . . . PTH bi dng hc sinh gii . . . . . . Trng THPT chuyn Quang Trung

You might also like